General 1 Flashcards

1
Q
1. A 55-year-old man has severe pain on gentle touching of the arm. Six months ago, the median nerve was damaged during creation of an arteriovenous fistula for dialysis. Which of the following terms best describes 
this phenomenon? 
(A) Allodynia 
(B) Hyperalgesia 
(C) Hyperpathia 
(D) Hypersensitivity 
(E) Hypesthesia
A

A

How well did you know this?
1
Not at all
2
3
4
5
Perfectly
2
Q
2. Which of the following receptors mediates the windup phenomenon? 
(A) 5-HT1D 
(B) Mu-opioid 
(C) Nicotinic 
(D) N-methyl-D-aspartic acid (NMDA) 
(E) Vanilloid
A

D

How well did you know this?
1
Not at all
2
3
4
5
Perfectly
3
Q
3. An 8-year-old girl is scheduled for a caudal injection for treatment of pain after a ureteral reimplantation. 
Which of the following ligaments will be traversed by the needle immediately prior to entering the caudal 
canal? 
(A) Interspinous 
(B) Multifidus 
(C) Sacroanal 
(D) Sacrococcygeal 
(E) Sacrospinous
A

D

How well did you know this?
1
Not at all
2
3
4
5
Perfectly
4
Q
  1. A 25-year-old janitor has severe back pain and is unable to stand up straight after he fell at work. On physical
    examination, he has positive pain behavior but no neurologic deficits. A surveillance video from the
    insurance company shows he continues to play baseball with friends. Which of the following is the most
    likely diagnosis?
    (A) Conversion disorder
    (B) Factitious disorder
    (C) Hypochondriasis
    (D) Malingering
    (E) Somatization disorder with psychological factors
A

D

How well did you know this?
1
Not at all
2
3
4
5
Perfectly
5
Q
  1. Which of the following conditions is most likely to result from prolonged use of high doses of meperidine to
    treat pain in a patient with renal failure?
    (A) Hyperthermia
    (B) Peripheral neuropathy
    (C) Progressive pruritus
    (D) Seizures
    (E) Tolerance to constipation
A

D

How well did you know this?
1
Not at all
2
3
4
5
Perfectly
6
Q
  1. Compared with an epidural dose of morphine, a single epidural dose of fentanyl produces which of the
    following?
    (A) Higher incidence of respiratory depression
    (B) Less segmental spread of analgesia
    (C) Longer duration of action
    (D) More intense pruritus
    (E) Slower onset of analgesia
A

B

How well did you know this?
1
Not at all
2
3
4
5
Perfectly
7
Q
  1. A 70-kg, 60-year-old man with hypertension and chronic renal failure is unresponsive and apneic. Two days
    ago, he began administration of patient-controlled analgesia (PCA) with morphine at a continuous rate of 1
    mg/hr with a dose of 2 mg and a lockout interval of 10 minutes. The dose of morphine totaled 60 mg over 48 hours. Which of the following is the most likely cause of the patient’s condition?
    (A) Accumulation of morphine-6-glucuronide
    (B) Accumulation of unmetabolized morphine
    (C) Brainstem ischemia caused by hypotension
    (D) Mechanical failure of the PCA device
    (E) Progressive uremia
A

A

How well did you know this?
1
Not at all
2
3
4
5
Perfectly
8
Q
  1. A 38-year-old woman with breast cancer is switched from oral morphine 90 mg every eight hours to morphine
    1 mg daily via percutaneous intrathecal catheter because of excessive sedation. Thirty-six hours later, she has
    confusion, diaphoresis, and tachycardia. Which of the following is the most likely cause of her symptoms?
    (A) Increased intracranial pressure
    (B) Meningitis
    (C) Metabolic abnormalities
    (D) Opioid withdrawal
    (E) Sepsis
A

D

How well did you know this?
1
Not at all
2
3
4
5
Perfectly
9
Q
  1. A 47-year-old man requires analgesia in the ICU after sustaining fractures of both femurs in a motor vehicle
    collision. He has developed hepatorenal syndrome. Which of the following drugs is most appropriate for pain
    control?
    (A) Buprenorphine
    (B) Codeine
    (C) Fentanyl
    (D) Meperidine
    (E) Morphine
A

C

How well did you know this?
1
Not at all
2
3
4
5
Perfectly
10
Q
  1. A 70-year-old man with early Parkinson’s disease is being treated with intravenous patient-controlled
    analgesia (PCA) with morphine following total knee arthroplasty. His pain is well controlled, but he has
    nausea. Which of the following is the most appropriate antiemetic for this patient?
    (A) Droperidol
    (B) Metoclopramide
    (C) Ondansetron
    (D) Prochlorperazine
    (E) Promethazine
A

C

How well did you know this?
1
Not at all
2
3
4
5
Perfectly
11
Q
  1. A 32-year-old woman is referred to a pain clinic to receive analgesic therapy for chronic, intractable low back
    pain. She has a 15-year history of major depressive disorder, in remission, for which she receives phenelzine.
    Which of the following medications is most likely to cause serotonin syndrome in this patient?
    (A) Gabapentin
    (B) Lidocaine
    (C) Morphine
    (D) Naproxen
    (E) Tramadol
A

E

How well did you know this?
1
Not at all
2
3
4
5
Perfectly
12
Q
  1. A 40-year-old patient has difficulty falling asleep at night due to chronic myofascial pain. Which of the
    following is most appropriate for the treatment of this patient?
    (A) Amitriptyline
    (B) Combination of butalbital, caffeine, aspirin
    (C) Combination of oxycodone and acetaminophen
    (D) Fluoxetine
    (E) Triazolam
A

A

How well did you know this?
1
Not at all
2
3
4
5
Perfectly
13
Q
  1. A 76-year-old woman has intense paroxysms of unilateral electric-shocklike pain in the face that is usually
    triggered by brushing her teeth or washing her face. Which of the following is the best initial management?
    (A) Administration of baclofen
    (B) Administration of carbamazepine
    (C) Microvascular decompression of the trigeminal nerve root
    (D) Radiofrequency lesioning of the gasserian ganglion
    (E) Trigeminal nerve block
A

B

How well did you know this?
1
Not at all
2
3
4
5
Perfectly
14
Q
  1. A 35-year-old woman who has had fibromyalgia for the past 10 years is undergoing evaluation with a new
    physician. She has been taking alprazolam daily to decrease muscle tension, emotional distress, and anxiety,
    but her prescription ran out four days ago. She is anxious, irritable, and tremulous. Examination shows
    tachycardia and diaphoresis. Which of the following is the most appropriate pharmacotherapy for this
    patient?
    (A) Buspirone
    (B) Diazepam
    (C) Flumazenil
    (D) Paroxetine
    (E) Propranolol
A

B

How well did you know this?
1
Not at all
2
3
4
5
Perfectly
15
Q
  1. A 62-year-old woman with chronic pain is referred to a pain specialist for consultation. The patient has been
    taking an undetermined amount of oxycodone for the past four months and the referring physician is
    concerned that the patient is addicted to pain medications. When the pain specialist goes to see the patient in
    the hospital room, her adult son is in the room. Which of the following is the most appropriate action by the
    physician in terms of conducting a patient interview?
    (A) Ask the patient whether she would like the son to stay during the interview
    (B) Ask the son to leave the room and proceed with the interview
    (C) Return later when the patient is alone
    (D) Tell the son that he can stay in the room as long as he does not interfere with the interview
A

A

How well did you know this?
1
Not at all
2
3
4
5
Perfectly
16
Q
  1. The ilioinguinal nerve lies
    (A) adjacent to the anterosuperior iliac spine
    (B) inferior to the inguinal ligament
    (C) medial to the femoral artery
    (D) superficial to the external oblique muscle
    (E) within the inguinal ring
A

A

How well did you know this?
1
Not at all
2
3
4
5
Perfectly
17
Q
  1. A 29-year-old man has had low back pain since falling from a ladder at work and sustaining a coccygeal
    fracture four months ago. His pain is constant and most severe with sitting. The pain is most likely to be
    blocked by local anesthetic injection at which of the following sites?
    (A) Anococcygeal ligament
    (B) Ganglion impar
    (C) Pudendal nerves
    (D) S4-5 nerve root
    (E) Superior hypogastric plexus
A

D

How well did you know this?
1
Not at all
2
3
4
5
Perfectly
18
Q
  1. A 27-year-old woman comes to the pain clinic because of pain in the shoulder and suprascapular region
    one month after undergoing modified radical neck dissection for papillary carcinoma of the thyroid.
    Examination shows decreased sensation to light touch over the superior aspect of the right trapezius and
    scapular winging with resisted shoulder abduction. Which of the following nerves was most likely transected
    during the procedure?
    (A) Axillary
    (B) Dorsal scapular
    (C) Long thoracic
    (D) Spinal accessory
    (E) Thoracodorsal
A

C

How well did you know this?
1
Not at all
2
3
4
5
Perfectly
19
Q
  1. A patient has palpitations, flushing, and headache after undergoing gingival injection of a local anesthetic.
    Which of the following is the most likely cause of this reaction?
    (A) Epinephrine in the local anesthetic
    (B) Local anesthetic allergy
    (C) Methylparaben reaction
    (D) Para-aminobenzoic acid allergy
    (E) Vasovagal reaction
A

A

How well did you know this?
1
Not at all
2
3
4
5
Perfectly
20
Q
  1. A 31-year-old man undergoes a right thoracotomy and bleb resection with combined general and thoracic
    epidural (T4-5) catheter anesthesia. Following the procedure, pain is controlled with an infusion of
    0.1% bupivacaine and fentanyl 5 μg/mL at 6 mL/hr via the epidural catheter. Twenty hours later, the patient
    has severe pain at the incision site. Blood pressure is 170/100 mmHg and heart rate is 97 bpm. The most
    appropriate management at this time is to
    (A) administer a bolus of 1.5% lidocaine 4 mL through the epidural catheter
    (B) administer 15 mg of ketorolac intravenously every six hours
    (C) begin intravenous patient-controlled analgesia with morphine
    (D) increase the concentration of bupivacaine to 0.2%
    (E) increase the epidural infusion rate of the bupivacaine-fentanyl solution to 8 mL/hr
A

A

How well did you know this?
1
Not at all
2
3
4
5
Perfectly
21
Q
  1. A lumbar sympathetic block is appropriate treatment for each of the following conditions of the lower
    extremity EXCEPT
    (A) Buerger’s disease
    (B) complex regional pain syndrome type I
    (C) frostbite
    (D) meralgia paresthetica
    (E) phantom limb pain
A

D

How well did you know this?
1
Not at all
2
3
4
5
Perfectly
22
Q
  1. A 42-year-old man has excellent relief of pain five days after undergoing a lumbar epidural injection of
    methylprednisolone 80 mg diluted to a volume of 10 mL with normal saline. Which of the following is the
    most common adverse effect of this treatment?
    (A) Arachnoiditis
    (B) Chemical meningitis
    (C) Headache
    (D) Hyperglycemia
A

D

How well did you know this?
1
Not at all
2
3
4
5
Perfectly
23
Q
  1. A 44-year-old woman has right hip pain that is reproduced by right leg abduction, external rotation, and
    palpation of the greater trochanter. Flexion and extension of the hip are not painful. The most appropriate
    interventional therapy for this patient is injection at which of the following sites?
    (A) Hip joint
    (B) Ischial bursa
    (C) Piriformis muscle
    (D) Trochanteric bursa
A

D

How well did you know this?
1
Not at all
2
3
4
5
Perfectly
24
Q
  1. A 30-year-old woman experiences painless shortness of breath 30 minutes after undergoing a stellate ganglion
    block with 10 mL of 2% lidocaine for treatment of complex regional pain syndrome type I involving the right
    arm. She had sustained a fracture of the right forearm five months ago. Which of the following is the most
    likely cause of this symptom?
    (A) Anaphylactic reaction to lidocaine
    (B) Hematoma
    (C) Neurogenic pulmonary edema
    (D) Phrenic nerve block
    (E) Recurrent laryngeal nerve block
A

D

How well did you know this?
1
Not at all
2
3
4
5
Perfectly
25
Q
  1. Blockade of the superficial cervical plexus is performed at which location?
    (A) At the midposterior border of the sternocleidomastoid muscle
    (B) In the interscalene groove
    (C) Over Chassaignac tubercle
    (D) Over the mastoid process
    (E) Over the transverse process of C2
A

A

How well did you know this?
1
Not at all
2
3
4
5
Perfectly
26
Q
26. One percent plain lidocaine 30 mL injected at which of the following sites results in the highest level of 
systemic absorption? 
(A) Brachial plexus 
(B) Caudal 
(C) Epidural 
(D) Intercostal 
(E) Subcutaneous
A

D

How well did you know this?
1
Not at all
2
3
4
5
Perfectly
27
Q
27. During stellate ganglion block, a patient becomes restless, tachypneic, and disoriented immediately after 
receiving the test dose of local anesthetic. Which of the following is the most likely cause of these 
symptoms? 
(A) Intravenous injection 
(B) Phrenic nerve paralysis 
(C) Pneumothorax 
(D) Subarachnoid block 
(E) Vertebral artery injection
A

E

How well did you know this?
1
Not at all
2
3
4
5
Perfectly
28
Q
  1. When using C-arm fluoroscopy to perform a therapeutic procedure, which of the following is the most
    effective method of preventing unnecessary radiation exposure to the patient?
    (A) Decreasing the tube distance to the patient
    (B) Increasing patient-to-image intensifier distance
    (C) Shielding the patient with a lead coverup
    (D) Using strict collimation of the beam
    (E) Using the shortest beam-on time possible
A

E

How well did you know this?
1
Not at all
2
3
4
5
Perfectly
29
Q
29. Which of the following is the most likely adverse effect of neurolytic celiac plexus block using 30 mL of 
10% phenol in glycerin? 
(A) Decreased gastrointestinal motility 
(B) Leg weakness 
(C) Orthostatic hypotension 
(D) Seizure 
(E) Urinary retention
A

C

How well did you know this?
1
Not at all
2
3
4
5
Perfectly
30
Q
  1. A 56-year-old man who has pancreatic carcinoma and midepigastric abdominal pain is referred for celiac
    plexus block. The needles are placed, and their position is confirmed with CT scan. After injection of the first
    2 mL of a 0.25% bupivacaine solution containing 1:200,000 epinephrine, the patient’s heart rate transiently
    increases from 68 bpm to 120 bpm. Which of the following is the most appropriate next step?
    (A) Administer midazolam 1 mg intravenously
    (B) Continue injection of 0.25% bupivacaine with 1:200,000 epinephrine in 3- to 5-mL increments
    (C) Remove the needles and cancel the procedure
    (D) Reposition the needles and repeat the injection of 2 mL of 0.25% bupivacaine with 1:200,000
    epinephrine
    (E) Wait until the heart rate returns to baseline before continuing the injection of 0.25% bupivacaine
    without epinephrine
    Picture: File PMExamSampleQuestions
A

D

How well did you know this?
1
Not at all
2
3
4
5
Perfectly
31
Q
  1. A 47-year-old woman has had low back pain for the past six weeks that has not responded to treatment with
    rest, oral analgesics, and physical therapy. MRI of the lumbosacral spine is shown. The image shown reveals
    which of the following?
    (A) Central herniated disk, L4-L5, without compromise of the neural foramina
    (B) Central herniated disk, L5-S1, without compromise of the neural foramina
    (C) Left-sided herniated disk, L4-L5, with compromise of the neural foramen
    (D) Left-sided herniated disk, L5-S1, with compromise of the neural foramen
    (E) Normal findings
A

C

How well did you know this?
1
Not at all
2
3
4
5
Perfectly
32
Q
  1. A 60-year-old woman with a chronic S1 radiculopathy has mild bilateral weakness of leg flexion, plantar and
    dorsiflexion at the ankle, and toe extension. An MRI four months ago showed a small paracentral herniated
    disk at the L4-5 level. Which of the following is the most appropriate first step in management?
    (A) Bilateral selective nerve root injection at the S1 neural foramina
    (B) Epidural corticosteroid injection at the L5-S1 level
    (C) Repeat MRI
    (D) Trial of back extension exercises
    (E) Trial spinal cord stimulation

Picture: File PMExamSampleQuestions

A

C

How well did you know this?
1
Not at all
2
3
4
5
Perfectly
33
Q
  1. A 35-year-old man is referred for evaluation of a six-month history of increasingly severe low back pain that
    began insidiously without any identifiable trauma. The pain is deep and aching over the low back. He has no
    pain in his extremities. A discrete lesion is shown on the accompanying MRI within the inferior endplate of
    the L4 vertebral body. Which of the following is the most appropriate next step?
    (A) Obtain a bone scan of the lumbosacral spine
    (B) Obtain a complete blood count
    (C) Obtain a CT scan of the lumbosacral spine
    (D) Obtain an MRI of the lumbosacral spine with gadolinium contrast enhancement
    (E) Proceed with initial treatment without further diagnostic evaluation
A

E

How well did you know this?
1
Not at all
2
3
4
5
Perfectly
34
Q
  1. Which of the following is associated with pain arising from the sacroiliac joint?
    (A) Paresthesia of the lateral thigh
    (B) Positive Patrick’s (FABER) test
    (C) Sexual dysfunction
    (D) Weakness of plantar flexion of the great toe
A

B

How well did you know this?
1
Not at all
2
3
4
5
Perfectly
35
Q
35. A 14-year-old girl has intermittent bitemporal and occipital headaches that are worse when she is at school. 
She has undergone counseling at school for depression and anxiety. Which of the following is the most likely 
diagnosis? 
(A) Cluster headache 
(B) Malingering 
(C) Migraine headache 
(D) Temporal arteritis 
(E) Tension-type headache
A

E

How well did you know this?
1
Not at all
2
3
4
5
Perfectly
36
Q
  1. A 60-year-old woman comes to the pain clinic because of a two-month history of fever as well as severe,
    persistent pain over the left side of the jaw and over the left ear. She also has loss of appetite with associated
    fatigue. Physical examination shows tenderness over the area above her left ear with mild swelling.
    Erythrocyte sedimentation rate is greater than 48 mm/hr. Which of the following is the most likely diagnosis?
    (A) Atypical complex regional pain syndrome
    (B) Postherpetic neuralgia
    (C) Temporal arteritis
    (D) Temporomandibular joint dysfunction
    (E) Trigeminal neuralgia
A

C

How well did you know this?
1
Not at all
2
3
4
5
Perfectly
37
Q
  1. A 24-year-old woman has had severe unilateral headaches since menarche. The headaches are preceded by
    flashing spots before her eyes, are accompanied by nausea with occasional vomiting, and last six to eight
    hours. The patient reports that she was recently promoted at her job and that her mother and maternal aunt
    had similar headaches when they were younger. She has been taking oral contraceptives. Although she
    appears anxious, physical examination is normal. Appropriate management for this patient should include
    each of the following EXCEPT
    (A) administration of acetaminophen with codeine
    (B) administration of sumatriptan
    (C) avoidance of caffeine-containing beverages
    (D) change of her oral contraceptive
    (E) maintenance of a strict sleeping schedule
A

A

How well did you know this?
1
Not at all
2
3
4
5
Perfectly
38
Q
  1. A 38-year-old man has had a persistent headache in the frontal and occipital areas bilaterally since undergoing
    diagnostic lumbar puncture one week ago. The pain is worse when he sits or stands and is relieved when he
    lies down. The patient is HIV positive and has candidal and cytomegaloviral infections. His CD4+
    lymphocyte count is 150/mm3
    . Which of the following is the most appropriate management of his headache?
    (A) Caudal blood patch with donor blood
    (B) Caudal blood patch with the patient’s blood
    (C) Lumbar blood patch with donor blood
    (D) Lumbar blood patch with the patient’s blood
    (E) Lumbar saline infusion
A

D

How well did you know this?
1
Not at all
2
3
4
5
Perfectly
39
Q
  1. A 25-year-old man has pain and paresthesias in the thigh two weeks after being involved in a head-on motor
    vehicle accident. Current examination shows decreased sensation in the thigh; there is no loss of muscle
    strength. Which of the following nerves is most likely injured?
    (A) Genitofemoral
    (B) Ilioinguinal
    (C) Lateral femoral cutaneous
    (D) Obturator
    (E) Saphenous
A

C

How well did you know this?
1
Not at all
2
3
4
5
Perfectly
40
Q
  1. A 28-year-old man has had constant pain in a bandlike pattern at the waistline since sustaining an incomplete
    transection of the spinal cord at T12 five months ago. Which of the following is the most likely explanation?
    (A) Central neuropathic
    (B) Mechanical instability
    (C) Primary myofascial pain
    (D) Psychosomatic pain
    (E) Referred visceral pain
A

A

How well did you know this?
1
Not at all
2
3
4
5
Perfectly
41
Q
41. Which of the following regions is most commonly affected in patients who have acute herpes zoster infection? 
(A) Cervical 
(B) Lumbar 
(C) Sacral 
(D) Thoracic 
(E) Trigeminal
A

D

How well did you know this?
1
Not at all
2
3
4
5
Perfectly
42
Q
  1. A 35-year-old man who has a 10-year history of HIV infection and intravenous drug use has recent onset of
    burning pain and dysesthesias in both feet. Nerve conduction studies show a primary sensory symmetric
    peripheral neuropathy. Administration of which of the following medications is the most appropriate initial
    step in management?
    (A) Dexamethasone
    (B) Etanercept
    (C) Methadone
    (D) Pregabalin
    (E) Tramadol
A

D

How well did you know this?
1
Not at all
2
3
4
5
Perfectly
43
Q
  1. A 43-year-old woman is being treated for painful neuropathy in both feet. She is HIV positive, which was
    caused by a blood transfusion two years ago. Her disease indices are stable, and she is taking appropriate
    antiviral agents. Acetaminophen/codeine preparations initially helped relieve her symptoms so she could
    continue to work as a security guard. Which of the following is the most appropriate management of the foot
    pain?
    (A) Discontinue codeine
    (B) Initiate antiepileptic therapy
    (C) Perform serial lumbar sympathetic nerve blocks
    (D) Perform transcutaneous electrical nerve stimulation
    (E) Prescribe orthotics
A

B

How well did you know this?
1
Not at all
2
3
4
5
Perfectly
44
Q
  1. A 65-year-old man has had aching in the legs and burning and tingling in the upper and lower extremities for
    the past two weeks. He has been taking isoniazid for four months for treatment of latent tuberculosis. Which
    of the following agents is most appropriate for initial treatment of this patient’s symptoms?
    (A) Amitriptyline
    (B) Clonidine
    (C) Gabapentin
    (D) Pyridoxine (vitamin B6)
    (E) Thiamine
A

D

How well did you know this?
1
Not at all
2
3
4
5
Perfectly
45
Q
  1. A 55-year-old woman comes to the pain clinic for a follow-up examination two months after sustaining a
    fracture of the left wrist. Initial treatment consisted of cast immobilization for six weeks. Since removal of the
    cast, there has been swelling, sensitivity to touch, erythema, and burning pain. Physical therapy exacerbates
    the symptoms. A trial of nortriptyline therapy has provided no relief of the pain. Which of the following is the
    most appropriate next step in management?
    (A) Initiate massage therapy
    (B) Initiate opioid therapy
    (C) Perform peripheral nerve block
    (D) Perform stellate ganglion block
    (E) No further management is necessary
A

D

How well did you know this?
1
Not at all
2
3
4
5
Perfectly
46
Q
  1. One of the effects created by activation or increased release of substance P is

(A) vasoconstriction.

(B) membrane stabilization.

(C) analgesia.

(D) vasodilati

A

D

How well did you know this?
1
Not at all
2
3
4
5
Perfectly
47
Q
  1. A 23-year-old female patient, who was recently discharged from the hospital following open reduction and internal fixation of a fractured femur, suddenly develops severe chest pain. Which of the following medications in her history would seem to be implicated in the etiology of her pain?

(A) Oral contraceptives

(B) Nonsteroidal anti-inflammatory agents

(C) Opioid analgesics

(D) Benzodiazepines

A

A

How well did you know this?
1
Not at all
2
3
4
5
Perfectly
48
Q
  1. Patients who present with fever and pain of recent onset over the neck, upper back, chest, and upper limbs should be assessed for the possibility of abscess in the

(A) cervical epidural space.

(B) posterior nasopharynx.

(C) subdiaphragmatic space.

(D) T 7-8 disk space.

A

A

How well did you know this?
1
Not at all
2
3
4
5
Perfectly
49
Q
  1. Disability due to chronic pain is felt to be primarily related to the

(A) number of somatic sites in which pain exists.

(B) reinforcement of pain behaviors.

(C) presence of a life-threatening disease.

(D) presence of neuropathic, as opposed to muscular, pain causes.

A

B

How well did you know this?
1
Not at all
2
3
4
5
Perfectly
50
Q
  1. Further testing with CT scan or MRI is mandatory

in headaches accompanied by all of the following EXCEPT

(A) prolonged long-term, unchanging band-like headaches.

(B) hemiparesis and contralateral sensory deficit.

(C) the appearance of seizures.

(D) olfactory hallucinations.

A

A

How well did you know this?
1
Not at all
2
3
4
5
Perfectly
51
Q
  1. Referral to a multidisciplinary pain center is usually most appropriate when patients demonstrate evidence of

(A) purely psychiatric mechanisms.

(B) purely neuropathic mechanisms.

(C) both psychological tension and physical muscle tension.

(D) both somatic and psychological factors.

A

D

How well did you know this?
1
Not at all
2
3
4
5
Perfectly
52
Q
  1. Which tricyclic antidepressant is most appropriate for treatment of pain in an 80-year-old male with postherpetic neuralgia and urinary retention?

(A) Amitriptyline (Elavil)

(B) Doxepin (Sinequan)

(C) Desipramine (Norpramin)

(D) Imipramine (Tofranil)

A

C

How well did you know this?
1
Not at all
2
3
4
5
Perfectly
53
Q
  1. In disability determination under most workers’ compensation systems, the presence of pain is given

(A) more attention than the underlying physical impairment.

(B) as much attention as the underlying physical impairment.

(C) less attention than the underlying physical

impairment.

(D) no attention whatsoever.

A

C

How well did you know this?
1
Not at all
2
3
4
5
Perfectly
54
Q
  1. Which of the following is true regarding the use of antidepressants to reduce chronic pain?

(A) Only tertiary amine tricyclics are effective.

(B) Serotonergic agents are not clearly superior to noradrenergic ones.

(C) Serotonin potentiation is a necessary characteristic of effective agents.

(D) Only noradrenergic agents are effective.

A

B

How well did you know this?
1
Not at all
2
3
4
5
Perfectly
55
Q
  1. DREZ lesions have been documented to provide long-term pain relief in

(A) cervical root avulsion.

(B) sciatica.

(C) diabetic neuropathy.

(D) thalamic pain syndrome.

A

A

How well did you know this?
1
Not at all
2
3
4
5
Perfectly
56
Q
  1. A 52-year-old patient presents with a history of acute low back pain, without trauma, which is unrelieved by bed rest and is associated with paroxysms of pain and an elevated erythrocyte sedimentation rate. Radiographs of the spine reveal an absent pedicle. The most likely diagnosis is

(A) lupus erythematosus.

(B) multiple myeloma.

(C) metastatic lesion.

(D) disc space infection.

A

C

How well did you know this?
1
Not at all
2
3
4
5
Perfectly
57
Q
  1. Which of the following is true regarding patients with cluster headaches?

(A) They are more likely to be female.

(B) They are likely to lie in a quiet, dark room with an ice pack over the affected temple during an attack.

(C) They are usually nonsmokers and nondrinkers.

(D) They are known to attempt suicide secondary to their pain.

A

D

How well did you know this?
1
Not at all
2
3
4
5
Perfectly
58
Q
  1. Which of the following is true of the physical or sensory component of pain perception?

(A) It is less variable than the anxiety produced by the pain.

(B) It is more variable than the anxiety produced by the pain.

(C) It is generally equal to the anxiety produced by the pain.

(D) It is reduced in patients with hypochondriasis.

A

A

How well did you know this?
1
Not at all
2
3
4
5
Perfectly
59
Q
  1. A patient who has been taking high doses of benzodiazepines and opioids experiences withdrawal symptoms during detoxification. Which of the following specifically indicates that the opioid is being tapered too rapidly?

(A) Hyperreflexia

(B) Diaphoresis

(C) Hyperactive bowel sounds

(D) Tachycardia

A

C

How well did you know this?
1
Not at all
2
3
4
5
Perfectly
60
Q
  1. The depression commonly seen in those with chronic pain of nonmalignant origin differs from the most typical major depressions in that in the former there is likely to be

(A) anhedonia.

(B) weight gain.

(C) guilty ruminations.

(D) insomnia.

A

B

How well did you know this?
1
Not at all
2
3
4
5
Perfectly
61
Q
  1. All of the following are true of migraine headache EXCEPT

(A) Aura (prodrome) is not present in common migraine.

(B) The neurologic symptoms of classic migraine may persist beyond the headache phase.

(C) Ergotamine (Ergostat) is effective in treating acute attacks when used daily for 7-14 days.

(D) 70% of migraine patients have a positive family migraine history.

A

C

How well did you know this?
1
Not at all
2
3
4
5
Perfectly
62
Q
  1. The essential feature of pain that can be used to differentiate it from other somatic sensations is its

(A) intensity.

(B) threshold.

(C) chronicity.

(D) unpleasantness.

A

D

How well did you know this?
1
Not at all
2
3
4
5
Perfectly
63
Q
  1. Aching pain in the suprapubic region is most likely caused by abnormalities of the

(A) ureter.

(B) prostate.

(C) coccyx.

(D) sacroiliac joints.

A

B

How well did you know this?
1
Not at all
2
3
4
5
Perfectly
64
Q

Question 1
A chronic pain client reports to you, the charge nurse, that the nurse have not been responding to requests for pain medication. What is your initial action?
A. Check the MARs and nurses’ notes for the past several days.
B. Ask the nurse educator to give an in-service about pain management.
C. Perform a complete pain assessment and history on the client.
D. Have a conference with the nurses responsible for the care of this client

A

D

How well did you know this?
1
Not at all
2
3
4
5
Perfectly
65
Q
Question 2
Family members are encouraging your client to “tough it out” rather than run the risk of becoming addicted to narcotics. The client is stoically abiding by the family’s wishes. Priority nursing interventions for this client should target which dimension of pain?
A. Sensory
B. Affective
C. Sociocultural
D. Behavioral
E. Cognitive
A

C

How well did you know this?
1
Not at all
2
3
4
5
Perfectly
66
Q
Question 3
A client with diabetic neuropathy reports a burning, electrical-type in the lower extremities that is not responding to NSAIDs. You anticipate that the physician will order which adjuvant medication for this type of pain?
A. Amitriptyline (Elavil)
B. Corticosteroids
C. Methylphenidate (Ritalin)
D. Lorazepam (Ativan)
A

A
Antidepressants such as amitriptyline can be given for diabetic neuropathy. Corticosteroids are for pain associated with inflammation. Methylphenidate is given to counteract sedation if the client is on opioids. Lorazepam is an anxiolytic.

How well did you know this?
1
Not at all
2
3
4
5
Perfectly
67
Q

Question 4
Which client is most likely to receive opioids for extended periods of time?
A. A client with fibromyalgia
B. A client with phantom limb pain
C. A client with progressive pancreatic cancer
D. A client with trigeminal neuralgia

A

C
Question 4 Explanation:
Cancer pain generally worsens with disease progression and the use of opioids is more generous. Fibromyalgia is more likely to be treated with non-opioid and adjuvant medications. Trigeminal neuralgia is treated with anti-seizure medications such as carbamazepine (Tegretol). Phantom limb pain usually subsides after ambulation begins.

How well did you know this?
1
Not at all
2
3
4
5
Perfectly
68
Q

Question 5
As the charge nurse, you are reviewing the charts of clients who were assigned to a newly graduated RN. The RN has correctly charted dose and time of medication, but there is no documentation regarding non-pharmaceutical measures. What action should you take first?
A. Make a note in the nurse’s file and continue to observe clinical performance
B. Refer the new nurse to the in-service education department.
C. Quiz the nurse about knowledge of pain management
D. Give praise for the correct dose and time and discuss the deficits in charting.

A

D

How well did you know this?
1
Not at all
2
3
4
5
Perfectly
69
Q

Question 6
In caring for a young child with pain, which assessment tool is the most useful?
A. Simple description pain intensity scale
B. 0-10 numeric pain scale
C. Faces pain-rating scale
D. McGill-Melzack pain questionnaire

A

C
The Faces pain rating scale (depicting smiling, neutral, frowning, crying, etc.) is appropriate for young children who may have difficulty describing pain or understanding the correlation of pain to numerical or verbal descriptors. The other tools require abstract reasoning abilities to make analogies and use of advanced vocabulary.

How well did you know this?
1
Not at all
2
3
4
5
Perfectly
70
Q

Question 7
In applying the principles of pain treatment, what is the first consideration?
A. Treatment is based on client goals.
B. A multidisciplinary approach is needed.
C. The client must be believed about perceptions of own pain.
D. Drug side effects must be prevented and managed

A

C

How well did you know this?
1
Not at all
2
3
4
5
Perfectly
71
Q
Question 8
Which route of administration is preferred if immediate analgesia and rapid titration are necessary?
A. Intraspinal
B. Patient-controlled analgesia (PCA)
C. Intravenous (IV)
D. Sublingual
A

C
the IV route is preferred as the fastest and most amenable to titration. A PCA bolus can be delivered; however, the pump will limit the dosage that can be delivered unless the parameters are changed. Intraspinal administration requires special catheter placement and there are more potential complications with this route. Sublingual is reasonably fast, but not a good route for titration, medication variety in this form is limited.

How well did you know this?
1
Not at all
2
3
4
5
Perfectly
72
Q

Question 9
When titrating an analgesic to manage pain, what is the priority goal?
A. Administer smallest dose that provides relief with the fewest side effects.
B. Titrate upward until the client is pain free.
C. Titrate downwards to prevent toxicity.
D. Ensure that the drug is adequate to meet the client’s subjective needs.

A

A
the goal is to control pain while minimizing side effects. For severe pain, the medication can be titrated upward until pain is controlled. Downward titration occurs when the pain begins to subside. Adequate dosing is important; however, the concept of controlled dosing applies more to potent vasoactive drugs.

How well did you know this?
1
Not at all
2
3
4
5
Perfectly
73
Q

Question 10
In educating clients about non-pharmaceutical alternatives, which topic could you delegate to an experienced LPN/LVN, who will function under your continued support and supervision?
A. Therapeutic touch
B. Use of heat and cold applications
C. Meditation
D. Transcutaneous electrical nerve stimulation (TENS)

A

B
Use of heat and cold applications is a standard therapy with guidelines for safe use and predictable outcomes, and an LPN/LVN will be implementing this therapy in the hospital, under the supervision of an RN. Therapeutic touch requires additional training and practice. Meditation is not acceptable to all clients and an assessment of spiritual beliefs should be conducted. Transcutaneous electrical stimulation is usually applied by a physical therapist.

How well did you know this?
1
Not at all
2
3
4
5
Perfectly
74
Q
Question 11
Place the examples of drugs in the order of usage according to the World Health Organization (WHO) analgesic ladder.   a. Morphine, hydromorphone, acetaminophen and lorazepam b. NSAIDs and corticosteroids c. Codeine, oxycodone and diphenhydramine
A. B, A, C
B. C, A, B
C. B, C, A
D. A, B, C
A

C
Step 1 includes non-opioids and adjuvant drugs. Step 2 includes opioids for mild pain plus Step 1 drugs and adjuvant drugs as needed. Step 3 includes opioids for severe pain (replacing Step 2 opioids) and continuing Step 1 drugs and adjuvant drugs as needed.

How well did you know this?
1
Not at all
2
3
4
5
Perfectly
75
Q

Question 12
Which client is at greater risk for respiratory depression while receiving opioids for analgesia?
A. An elderly chronic pain client with a hip fracture
B. A client with a heroin addiction and back pain
C. A young female client with advanced multiple myeloma
D. A child with an arm fracture and cystic fibrosis

A

D
at greatest risk are elderly clients, opiate naïve clients, and those with underlying pulmonary disease. The child has two of the three risk factors

How well did you know this?
1
Not at all
2
3
4
5
Perfectly
76
Q

Question 13
A client appears upset and tearful, but denies pain and refuses pain medication, because “my sibling is a drug addict and has ruined out lives.” What is the priority intervention for this client?
A. Encourage expression of fears on past experiences
B. Provide accurate information about use of pain medication
C. Explain that addiction is unlikely among acute care clients
D. Seek family assistance in resolving this problem

A

A
This client has strong beliefs and emotions related to the issue of sibling addiction. First, encourage expression. This indicated to the client that the feelings are real and valid. It is also an opportunity to assess beliefs and fears. Giving facts and information is appropriate at the right time. Family involvement is important, bearing in mind that their beliefs about drug addiction may be similar to those of the client.

How well did you know this?
1
Not at all
2
3
4
5
Perfectly
77
Q
Question 14
A client is being tapered off opioids and the nurse is watchful for signs of withdrawal. What is one of the first signs of withdrawal?
A. Fever
B. Nausea
C. Diaphoresis
D. Abdominal cramps
A

C
Diaphoresis is one of the early signs that occur between 6 and 12 hours. Fever, nausea, and abdominal cramps are late signs that occur between 48 and 72 hours.

How well did you know this?
1
Not at all
2
3
4
5
Perfectly
78
Q

Question 15
In caring for clients with pain and discomfort, which task is most appropriate to delegate to the nursing assistant?
A. Assist the client with preparation of a sitz bath.
B. Monitor the client for signs of discomfort while ambulating
C. Coach the client to deep breathe during painful procedures
D. Evaluate relief after applying a cold application

A

A
The nursing assistant is able to assist the client with hygiene issues and knows the principles of safety and comfort for this procedure. Monitoring the client, teaching techniques, and evaluating outcomes are nursing responsibilities.

How well did you know this?
1
Not at all
2
3
4
5
Perfectly
79
Q

Question 16
The physician has ordered a placebo for a chronic pain client. You are newly hired nurse and you feel very uncomfortable administering the medication. What is the first action that you should take?
A. Prepare the medication and hand it to the physician
B. Check the hospital policy regarding use of the placebo.
C. Follow a personal code of ethics and refuse to give it.
D. Contact the charge nurse for advice

A

D

How well did you know this?
1
Not at all
2
3
4
5
Perfectly
80
Q

Question 17
For a cognitively impaired client who cannot accurately report pain, what is the first action that you should take?
A. Closely assess for nonverbal signs such as grimacing or rocking.
B. Obtain baseline behavioral indicators from family members.
C. Look at the MAR and chart, to note the time of the last dose and response.
D. Give the maximum PRS dose within the minimum time frame for relief.

A

B
Complete information from the family should be obtained during the initial comprehensive history and assessment. If this information is not obtained, the nursing staff will have to rely on observation of nonverbal behavior and careful documentation to determine pain and relief patterns

How well did you know this?
1
Not at all
2
3
4
5
Perfectly
81
Q
Which route of administration is preferable for administration of daily analgesics (if all body systems are functional)?
A. IV
B. IM or subcutaneous
C. Oral
D. Transdermal
E. PCA
A

C
If the gastrointestinal system is function, the oral route is preferred for routine analgesics because of lower cost and ease of administration. Oral route is also less painful and less invasive than the IV, IM, subcutaneous, or PCA routes. Transdermal route is slower and medication availability is limited compared to oral forms.

How well did you know this?
1
Not at all
2
3
4
5
Perfectly
82
Q

Question 19
A first day postoperative client on a PCA pump reports that the pain control is inadequate. What is the first action you should take?
A. Deliver the bolus dose per standing order.
B. Contact the physician to increase the dose.
C. Try non-pharmacological comfort measures.
D. Assess the pain for location, quality, and intensity

A

D
Assess the pain for changes in location, quality, and intensity, as well as changes in response to medication. This assessment will guide the next steps.

How well did you know this?
1
Not at all
2
3
4
5
Perfectly
83
Q

Question 20
Which non-pharmacological measure is particularly useful for a client with acute pancreatitis
A. Diversional therapy, such as playing cards or board games
B. Massage of back and neck with warmed lotion
C. Side-lying position with knees to chest and pillow against abdomen
D. Transcutaneous electrical nerve stimulation (TENS)

A

C
The side-lying, knee-chest position opens retroperitoneal space and provides relief. The pillow provides a splinting action. Diversional therapy is not the best choice for acute pain, especially if the activity requires concentration. TENS is more appropriate for chronic muscular pain. The additional stimulation of massage may be distressing to the client.

How well did you know this?
1
Not at all
2
3
4
5
Perfectly
84
Q
Question 21
What is the best way to schedule medication for a client with constant pain?
A. PRN at the client’s request
B. Prior to painful procedures
C. IV bolus after pain assessment
D. Around-the-clock
A

D
IF the pain is constant, the best schedule is around-the-clock, to provide steady analgesia and pain control. The other options may actually require higher doses to achieve control

How well did you know this?
1
Not at all
2
3
4
5
Perfectly
85
Q

Question 22
Which client(s) are appropriate to assign to the LPN/LVN, who will function under the supervision of the RN or team leader? (Choose all that apply.)
A. A client who needs pre-op teaching for use of a PCA pump
B. A client with a leg cast who needs neurologic checks and PRN hydrocodone
C. A client post-op toe amputation with diabetic neuropathic pain
D. A client with terminal cancer and severe pain who is refusing medication

A

B/C
Question 22 Explanation:
The clients with the cast and the toe amputation are stable clients and need ongoing assessment and pain management that are within the scope of practice for an LPN/LVN under the supervision of an RN. The RN should take responsibility for pre-operative teaching, and the terminal cancer needs a comprehensive assessment to determine the reason for refusal of medication.

How well did you know this?
1
Not at all
2
3
4
5
Perfectly
86
Q
Question 23
For a client who is taking aspirin, which laboratory value should be reported to the physician?
A. Potassium 3.6 mEq/L
B. Hematocrit 41%
C. PT 14 seconds
D. BUN 20 mg/dL
A

C
When a client takes aspirin, monitor for increases in PT (normal range 11.0-12.5 seconds in 85%-100%). Also monitor for possible decreases in potassium (normal range 3.5-5.0 mEq/L). If bleeding signs are noted, hematocrit should be monitored (normal range male 42%-52%, female 37%-47%). An elevated BUN could be seen if the client is having chronic gastrointestinal bleeding (normal range 10-20 mg/dL)

How well did you know this?
1
Not at all
2
3
4
5
Perfectly
87
Q

Question 24
Which client(s) would be appropriate to assign to a newly graduated RN, who has recently completed orientation? (Choose all that apply.)
A. An anxious, chronic pain client who frequently uses the call button
B. A client second day post-op who needs pain medication prior to dressing changes
C. A client with HIV who reports headache and abdominal and pleuritic chest pain
D. A client who is being discharged with a surgically implanted catheter

A

B
A second day postoperative client who needs medication prior to dressing changes has predictable and routine care that a new nurse can manage. Although chronic pain clients can be relatively stable, the interaction with this client will be time consuming and may cause the new nurse to fall behind. The HIV client has complex complaints that require expert assessment skills. The client pending discharge will need special and detailed instructions.

How well did you know this?
1
Not at all
2
3
4
5
Perfectly
88
Q

Question 25
A family member asks you, “Why can’t you give more medicine? He is still having a lot of pain.” What is your best response?
A. “The doctor ordered the medicine to be given every 4 hours.”
B. “If the medication is given too frequently he could suffer ill effects.”
C. “Please tell him that I will be right there to check of him.”
D. “Let’s wait about 30-40 minutes. If there is no relief I’ll call the doctor.”

A

C
directly ask the client about the pain and do a complete pain assessment. This information will determine which action to take next.

How well did you know this?
1
Not at all
2
3
4
5
Perfectly
89
Q
  1. A patient presents with muscle weakness. To assess his condition, you test his knee-jerk reflex
    by tapping his patella tendon with your hammer. Next you examine the jaw-jerk reflex by
    tapping his lower jaw with your finger. The pathways that mediate these two reflexes are
    similar in that both involve:
    A. dorsal root ganglia
    B. a 2-neuron, sensory to motor pathway
    C. pathways to the cortex through ventral posterior lateral nuclei
    D. cell bodies of sensory neurons in peripheral ganglia
    E. nuclei in the brainstem
A

B

How well did you know this?
1
Not at all
2
3
4
5
Perfectly
90
Q
  1. A 23 year old woman has sudden onset of fever, headache, and confusion lasting three days.
    Her physical exam shows disorientation. A spinal tap yields a CSF that is pink and DNA
    analysis indicates HSV infection. Considering the part of her brain that is most likely
    affected by the virus, which symptom could she exhibit from this condition?
    A. loss of tactile sensation
    B. astereognosis
    C. upper homonymous quandrantanopia
    D. inability to localize pain stimuli
    E. loss of accommodation reflex
A

C

How well did you know this?
1
Not at all
2
3
4
5
Perfectly
91
Q
  1. While I was cooking a few days ago, I reached into the oven and the top of my hand touched
    the heating element. My hand immediately withdrew, but the contact caused a burn on the top
    of my finger. The next day my finger was red and swollen and when I washed my hands in
    slightly warm water (30°C), the area around my burned finger became painful. Which of the
    following molecules contributed to the redness, swelling, and pain?
    A. glutamate
    B. substance P
    C. enkephalin
    D. norepinephrine
    E. acetylcholine
A

B

How well did you know this?
1
Not at all
2
3
4
5
Perfectly
92
Q
  1. A 45 year old man with an 8 year history of HIV sees his physician because he has had
    difficulty at his job as an accountant. He tells his physician that he was always good with math,
    but now he finds calculation difficult, he has trouble reasoning and he gets confused easily if
    there is more than one event occurring. His physician should schedule:
    A. an MRI
    B. a CT
    C. an X ray
    D. a lumbar puncture
    E.
A

A

How well did you know this?
1
Not at all
2
3
4
5
Perfectly
93
Q
5. A 14 year-old girl with recent onset of headaches, nausea, and cognitive difficulties is 
diagnosed with communicating hydrocephalus. The most likely location to cause this condition 
is:
A. interventricular foramina
B. cerebral aqueduct
C. foramen of Magendie
D. foramina of Lutschka
E. arachnoid granulations
A

E

How well did you know this?
1
Not at all
2
3
4
5
Perfectly
94
Q
6. An infant was born with a severe defect in which the top of its skull had not formed and its 
brain had largely failed to develop. This defect most likely began during which of the 
following periods:
A. week 1 of development
B. 3-4 weeks of development
C. 6-8 weeks of development
D. 3-4 months of development
E. 6 months of development
A

B

How well did you know this?
1
Not at all
2
3
4
5
Perfectly
95
Q
  1. A 29 year-old obese woman sees her physician because of a recent increase in headaches. On
    questioning she indicates that she has no history of headaches. An exam indicates papilledema,
    but a CT and MRI show no abnormalities. A likely reason for this woman’s symptoms is:
    A. subdural hemorrhage
    B. meningitis
    C. subarachnoid hemorrhage
    D. pseudotumor cerebri
    E. hydrocephalus
A

D

How well did you know this?
1
Not at all
2
3
4
5
Perfectly
96
Q
8. A 23 year-old woman is found comatose in her hotel room with an empty syringe next to her. 
Her respiration is slow and shallow and her pupils are miotic. Which of the following drugs 
could she NOT have taken:
A. morphine
B. fentanyl
C. buprenorphine
D. pentazocine
E. thebaine
A

E

How well did you know this?
1
Not at all
2
3
4
5
Perfectly
97
Q
  1. Which of the following choices characterizes the ionic changes in an axon that produce the
    action potential:
    A. influx of sodium and outflux of potassium
    B. influx of calcium and outflux of sodium
    C. influx of potassium and outflux of sodium
    D. influx of calcium and outflux of sodium
    E. influx of both sodium and potassium
A

A

How well did you know this?
1
Not at all
2
3
4
5
Perfectly
98
Q
  1. In an experiment in lab, you stimulate an axon once and then measure the size of the
    postsynaptic potential. You repeat the experiment, but this time you stimulate the axon at high
    frequency. When you measure the postsynaptic potential again, you find it is significantly
    larger in amplitude for many hours. The increased amplitude of the postsynaptic potential is
    due to
    A. an increased number of NMDA receptors
    B. an increased number of AMPA receptors
    C. a decreased number of NMDA receptors
    D. a decrease in sodium channel inactivation
    E. a decrease in potassium channel activation
A

B

How well did you know this?
1
Not at all
2
3
4
5
Perfectly
99
Q
  1. A 28 year-old man was shot in the back outside a bar during an argument. He was rushed to
    the ED where an exam revealed that he had no pin prick sensation in his left lower extremity.
    Vibratory sensation was intact on his body and limbs. The most likely location of a lesion
    causing these symptoms is:
    A. T6
    B. T8
    C. T12
    D. L2
    E. L4
A

C

How well did you know this?
1
Not at all
2
3
4
5
Perfectly
100
Q
  1. A 34 year-old woman gives birth to a baby boy whose head is noticeably enlarged. An MRI
    indicates incomplete formation of the foramina of Magendie and Luschka. An early
    developmental defect that might have resulted in malformation of these structures would have
    involved the:
    A. diencephalon
    B. mesencephalon
    C. telencephalon
    D. rhombencephalon
    E. prosencephalon
A

D

How well did you know this?
1
Not at all
2
3
4
5
Perfectly
101
Q
  1. A 30 year old woman who has been taking pain medication on the same prescription for
    several months complains to her doctor that her pain seems worse than it has been in the past.
    After examining the woman, her physician finds no change in the woman’s physical condition.
    The woman’s increased pain is an example of drug:
    A. adverse effects
    B. withdrawal
    C. tolerance
    D. dependence
    E. interaction
A

C

How well did you know this?
1
Not at all
2
3
4
5
Perfectly
102
Q
  1. A patient comes to you because she has difficulty seeing. You suspect a problem with her
    visual system. You would like to perform a noninvasive procedure to test its function, so you
    schedule her for an electroretinogram. This tests relies on which of the following properties:
    A. probability of Na+
    channel activation
    B. synchronized activity of many neurons
    C. efficiency of K+
    channel activation
    D. conduction velocity
    E. action potential amplitude
A

B

103
Q
15. The condition shown in the figure (myelin stain) involves 
similar sensory deficits to which of the syndromes below?
A. tabes dorsalis 
B. syringomyelia
C. Brown-Sequard
D. thalamic pain syndrome
E. Horner’s syndrome
Picture: Ex1-USMLEPrac
A

A

104
Q
  1. An infarct in a 65 year-old woman destroyed the VPL completely on the right side. Which of
    the following deficits would be associated with is lesion:
    A. loss of tactile sense on the left side of the face
    B. inability to feel pain on the left side of the body
    C. inability to feel pain on the left side of the face
    D. inability to localize pain on the left side of the body
    E. inability to localize pain on the left side of the face
A

D

105
Q
  1. A 39 year-old woman with fever of 102 ° F, headache, and neck stiffness was brought to the
    ED by her friend. She was worked up for possible meningitis, which was negative. Two days
    after she returned home, she called her physician to complain of a persistent headache, which
    worsened when she stood up but subsided when she reclined. The most likely reason for the
    headache is:
    A. migraine
    B. mis-diagnosed meningitis
    C. leakage of CSF
    D. subarachnoid hemorrhage
    E. subdural hemorrhage
A

C

106
Q
  1. A 60 year-old woman experiences sharp, shooting pain in her left oral cavity. The attacks are
    precipitated by brushing her teeth and cold fluids in her mouth. Between attacks she has no
    symptoms. The most likely treatment for this condition would be:
    A. Tylenol with codeine
    B. carbamazepine
    C. triptans
    D. oxycodone
    E. SSRI
A

B

107
Q
  1. A 70 year-old woman admitted to the ED is sent for imaging and her
    scan is shown in the figure. Based on the image, this woman has:
    A. right sided edema
    B. communicating hydrocephalus
    C. uncal herniation
    D. epidural hemorrhage
    E. hydrocephalus ex vacuo

Picture: Ex1-USMLEPrac

A

A

108
Q
  1. A 50 year-old man was brought to the ED by his partner because he exhibited signs of a
    stroke. A CT showed an infarcted area in the lateral aspect of the right frontal lobe. Which
    symptom would this man exhibit:
    A. left eye turned to the right
    B. right eye turned to the left
    C. dilated pupil in the left eye
    D. constricted pupil in the left eye
    E. left and right eyes turned to the right
A

E

109
Q
  1. A 31 year-old man visited his physician because he experienced headache accompanied by
    right-sided facial pain for the past few days. A neurological exam showed he also had a loss of
    tactile sensation on the left side of his body and face. An MRI indicated dissection of the
    carotid artery on the right side. Which other symptom is this man likely to have:
    A. dilated pupil on the right side
    B. ptosis and miosis on the right side
    C. intense facial pain triggered by touching the face
    D. nystagmus
    E. Lt and Rt eyes in fixed gaze to the left
A

B

110
Q
22. Arterial rupture was the likely cause of the recent death of a 14-year old boy who suffered a 
fatal blow to the temporal region of the skull while he was riding his skateboard. The death 
most likely would have resulted from: 
A. communicating hydrocephalus
B. epidural hematoma
C. subdural hematoma
D. subarachnoid hemorrhage
E. non-communicating hydrocephalus
A

B

111
Q
  1. A wife was brought to the ED by her husband after she complained about numbness on her
    face. A neurological exam showed loss of pain and temperature sensation on the RIGHT face
    and the LEFT body. A lesion causing these symptoms would be located in:
    A. right postcentral gyrus
    B. medial part of the rostral medulla on the right
    C. lateral part of the rostral medulla on the right
    D. right trigeminothalamic tract
    E. lateral part of the rostral pons on the left
A

C

112
Q
  1. A man fell off a ladder while working on his house and was rushed to the ED. An MRI
    showed damage at the level of T12 on the LEFT side in the anterolateral quadrant of his
    spinal cord. This injury would result in a loss of some sensation:
    A. from the entire leg on the right side
    B. from the entire leg on the left side
    C. from the knee down on the right side
    D. from the umbilicus down on the right side
    E. from the umbilicus down on the left side
A

A

113
Q
  1. A radiologist discussed a case with you in which a patient had a stroke involving the
    brainstem. The imaging showed a lesion area including the ventral trigeminothalamic tract at
    the level of the midbrain on one side. What symptoms would this patient show?
    A. loss of only pain and temperature sensation on the ipsilateral face
    B. loss of pain, temperature, and touch sensation on the ipsilateral face
    C. loss of pain, temperature, and touch sensation on the contralateral face
    D. paralysis of jaw muscles on the contralateral side
    E. loss of only touch sensation on the contralateral face
A

C

114
Q
  1. A patient displayed loss of pain, temperature and tactile sensation all on the left side of the
    body. These symptoms could be caused by a unilateral lesion in all of the following locations
    EXCEPT:
    A. spinal cord
    B. medulla
    C. pons
    D. midbrain
    E. internal capsule
A

A

115
Q
  1. A patient who has loss of pin prick sensation starting at the level of the nipple and downward
    on the right side would have a lesion in the spinal cord at which level:
    A. C7-C8
    B. T2-T3
    C. T4-T5
    D. T6-T7
    E. T9-T10
A

B

116
Q
  1. Damage to the area indicated in the figure, would result in :
    A. loss of pain and temperature sensation on the
    ipsilateral face
    B. loss of pain and temperature sensation on the
    contralateral face
    C. loss of pain, temperature, and touch sensation on the
    contralateral face
    D. loss of only touch sensation on the ipsilateral face
    E. loss of pain sensation on the contralateral body

Picture: Ex1-USMLEPrac

A

A

117
Q
  1. A patient displays a symmetrical loss of pain and temperature on
    the shoulder area on both sides of the body with no loss of tactile sensation. Pain and
    temperature and tactile sensation are normal over the rest of the body. This condition would be
    due mostly likely to:
    A. syringomyelia
    B. Brown-Sequard syndrome
    C. vitamin B12 deficiency
    D. Arnold Chiari syndrome
    E. tabes dorsalis
A

A

118
Q
  1. A patient shows a loss of tactile sensation on the right starting at the lateral aspect of the
    thigh and continuing over the rest of the lower right limb. There is also a loss of pain and
    temperature on the left from the medial aspect of the leg and below on the left. This would be
    caused by a lesion of the spinal cord:
    A. on the left at T12
    B. on the left at L2
    C. on the right at T12
    D. on the right at L2
    E. on the right at L4
A

D

119
Q
  1. As you examine a patient in the ED, you find she has loss of pain, temperature, and touch
    sensation on the right side of the face. Sensation on the rest of the body and face are normal.
    These symptoms could be caused by a lesion of the:
    A. left spinal trigeminal nucleus
    B. right spinal trigeminal nucleus
    C. right chief sensory nucleus of V
    D. ventral trigeminothalamic tract on the right
    E. ventral trigeminothalamic tract on the left
A

E

120
Q
  1. In a patient you are evaluating, you find an early-stage, extramedullary tumor growing on the
    ventrolateral surface of the spinal cord. The tumor is irritating nerve fibers adjacent to it and
    stimulating them. Considering the dermatomes affected by this tumor, which symptom would
    the patient experience:
    A. tingling sensation on one side of the body
    B. pain in the shoulders on one side
    C. loss of position sense in the leg on one side
    D. pain in the back of the leg on one side
    E. suspended, bilateral loss of pain sensation in the lower body
A

D

121
Q
  1. A patient has been diagnosed with a brainstem tumor involving cranial nerve VII on the right
    side. Which response would you expect when the right cornea is touched with a cotton
    applicator:
    A. no eyes blink
    B. right eye blinks
    C. left eye blinks
    D. both eyes blink
A

C

122
Q
  1. A patient was referred to you by another physician who was unable to diagnosis the patient’s
    disorder. When you test the patient in your office, you find that the patient cannot localize the
    source of a pin prick stimulus, but he can still feel the pin prick and reports it as feeling dull.
    This patient most likely has a lesion in:
    A. ALS
    B. substantia gelatinosa
    C. cingulate gyrus
    D. medial lemniscus
    E. postcentral gyrus
A

E

123
Q
35. A patient who has lost pain and temperature sensation from the level of the right little finger 
and the rest of the body below on the right side would most likely be caused by a lesion of 
the:
A. left ALS at C6
B. left ALS at T2
C. right ALS at C6
D. right ALS at C8
E. anterior white commissure at C8
A

A

124
Q
36. A patient shows loss of tactile sensation on the right side of the face, but pin prick sensation 
for the face and body are normal as well as tactile sensation for the body. This patient most 
likely has a lesion in the: 
A. spinal cord at C1
B. medulla
C. pons
D. midbrain
E. cortex
A

C

125
Q
  1. You are testing a patient’s pupillary light reflex and observe the following situation. When a
    light is shined into the left eye, only a direct response is obtained. However, when a light is
    shined into the right eye, only a consensual response is obtained. This is an indication that:
    A. There is a lesion of the right efferent pathway (CN III)
    B. There is a lesion of the left efferent pathway (CN III)
    C. There is a lesion of the right optic nerve
    D. There is a lesion of the left optic nerve
    E. There is a lesion in the right pretectal area
A

A

126
Q
  1. A man was shot in a hunting accident. In the ED it was determined that the bullet hit the ALS
    at the level of T2 on the right side. This lesion would cause:
    A. loss of pain and temp at the level of C8 on the left
    B. loss of pain and temp at the level of T4 on the left
    C. loss of pain and temp at the level of C8 on the right
    D. loss of tactile sensation at the level ofT2 on the right
    E. loss of tactile sensation at the level ofT2 on the left
A

B

127
Q
  1. You are involved in testing a new environmental pollutant found to be a neurotoxin. After
    injecting the agent into the brain, you discover that the levels of tyrosine hydroxylase
    decrease dramatically. Which of the following neurons would be impaired by this
    neurotoxin?
    A. dopaminergic and noradrenergic
    B. GABAminergic and glutaminergic
    C. serotonergic and dopaminergic
    D. serotonergic and glutaminergic
    E. serotonergic and GABAminergic
A

A

128
Q
  1. A 50 year-old man who comes to your office tends to slap his feet down on the floor as he
    walks toward you. On examination you find that he cannot tell the position of his legs when
    you move them passively and that there is decreased tactile and vibratory sensation in both
    lower limbs. Sensation to pin prick is normal. The most likely reason for these symptoms is:
    A. Syringomyelia
    B. Brown-Sequard Syndrome
    C. Tic Douloureux
    D. Tabes Dorsalis
    E. Myasthenia Gravis
A

D

129
Q
  1. Which of the following patients most likely has cluster headaches?
    A. 24 year old female with a 10 year history of unilateral throbbing headaches with
    associated nausea and vomiting. The patient has headaches every month right before
    her menstrual period
    B. 47 year old female with bilateral non-throbbing headaches that begin around noon
    every day and peak in the late afternoon
    C. 60 year old male who gets flurries of brief “electric shock” pains in his right lower
    face triggered by chewing
    D. 30 year old female with headaches lasting 2-4 days after receiving epidural anesthesia
    for the delivery of each of her four children
    E. 25 year old male with a two month history of severe non-throbbing left fronto-orbital
    headaches which last 30-40 minutes and awaken him from sleep in the early morning
    on a nearly daily basis. He had similar headaches for 10 weeks, 3 years ago.
A

E

130
Q
  1. On examination, you find that a patient’s eyes have the position at
    rest shown on the right. This is most likely due to a lesion of:
    A. Right cranial nerve VI nucleus
    B. Left cranial nerve VI nucleus
    C. MLF
    D. Left cranial nerve III
    E. Right cranial nerve III

Picture: Ex1-USMLEPrac

A

E

131
Q
  1. On examining a patient you obtain the following findings: Shining a light into the right eye
    produces no effect, but shining a light into the left eye causes both the left and right pupils to
    constrict. This result could be caused by a lesion of:
    A. right optic tract
    B. right optic nerve
    C. right cranial nerve III
    D. left cranial nerve III
    E. right pretectal area
A

B

132
Q
  1. A 30-year-old woman is unable to localize sound presented to her, but her hearing is normal
    otherwise. Which of the following structures would NOT be the location of a lesion causing this
    symptom:
    A. auditory cortex
    B. cochlear nucleus
    C. inferior colliculus
    D. lateral lemniscus
    E. medial geniculate nucleus
A

B

133
Q
  1. A 45 year old female gives a history of intermittent unilateral throbbing headaches with
    associated nausea and photophobia since age 20. She is currently having headaches 2-3 times
    a week lasting 4-8 hours. She has a history of hypertension. The most appropriate medication
    for treatment of her symptoms when they occur is:
    A. an ergot preparation
    B. anticonvulsant
    C. a combination preparation containing butalbital, caffeine and aspirin
    D. tricyclic antidepressant
    E. calcium channel blocker
A

C

134
Q
  1. A 30-year-old man presents with sudden-onset posterior
    hip pain and low back pain that began while he
    was moving heavy furniture. Which of the following
    maneuvers is most helpful in eliciting the most likely
    diagnosis?
    (A) Internal and external rotation of the hip
    (B) Ober’s test
    (C) Patrick’s maneuver
    (D) Straight leg raising test
A
  1. (D) Straight leg raising test. This patient likely has a
    compressed nerve. The straight leg raising test is used
    to elicit nerve impingement from a herniated disk in
    the lumbar spine, which frequently results in sciatica
    or posterior hip pain. Internal and external rotation
    of the hip and Patrick’s maneuver both elicit causes
    of anterior hip pain. Ober’s test is used to detect iliotibial
    band syndrome, a cause of lateral hip pain.
135
Q
  1. A 70-year-old woman presents with a 5-day history of
    worsening right hip pain. The pain has progressed
    to the point that she can no longer walk. The patient
    denies any history of recent trauma. Patrick’s maneuver
    is markedly positive. A plain radiograph of
    the hip shows no abnormalities, and magnetic resonance
    imaging (MRI) is ordered (Figure 1). What is
    this patient’s most likely diagnosis?
    (A) Avascular necrosis (AVN)
    (B) Hip fracture
    (C) Osteoarthritis
    (D) Trochanteric bursitis

Picture: hp_aug07_rqhip (1)

A
  1. (A) AVN. In Figure 1, decreased signal intensity is
    observed in the femoral head (wedge shape), which
    strongly suggests AVN. MRI is the most sensitive test
    for detecting AVN. Plain radiographs can be negative
    in early AVN. There is no evidence of hip fracture on
    this patient’s MRI, and she had no history of recent
    trauma. Osteoarthritis has a more insidious course
    and would not progress this rapidly (this patient’s hip
    pain developed over 5 days). Trochanteric bursitis
    causes lateral hip pain; Patrick’s maneuver would not
    be positive in trochanteric bursitis
136
Q
  1. A 70-year-old woman presents with acute right hip
    pain that started after a fall this morning. She is unable
    to walk because of severe pain. The patient has
    a history of osteoporosis for which she takes alendronate
    70 mg weekly. Patrick’s maneuver is markedly
    positive. A plain radiograph is obtained (Figure 2).
    What is this patient’s most likely diagnosis?
    (A) AVN
    (B) Hip fracture
    (C) Osteoarthritis
    (D) Trochanteric bursitis

Picture: hp_aug07_rqhip (1)

A
  1. (B) Hip fracture. This patient likely has a right hip
    fracture. In Figure 2, note the displaced femoral
    neck. Plain radiography is a good initial study but
    is not sensitive for diagnosing acute nondisplaced
    fractures. If this patient’s initial radiograph did not
    show hip fracture, further studies would be indicated
    given this patient’s recent fall, inability to walk,
    and history of osteoporosis. Computed tomography
    or MRI likely would reveal a nondisplaced acute
    fracture. Osteoarthritis has a more insidious course
    and would not cause acute pain. Trochanteric bursitis
    causes lateral hip pain
137
Q
  1. A 70-year-old man presents with progressive right
    hip pain for the past 2 months. He is diagnosed
    with osteoarthritis of the hip based on radiographic
    findings (Figure 3), and hip replacement surgery is recommended. A preoperative chest radiograph
    reveals a 3-cm right lung mass (Figure 4). Further
    history reveals that the patient is a nonsmoker. A
    biopsy of the hip reveals malignancy. What is this
    patient’s most likely diagnosis?
    (A) Adenocarcinoma
    (B) Large cell carcinoma
    (C) Small cell carcinoma
    (D) Squamous cell carcinoma

Picture: hp_aug07_rqhip (1)

A
  1. (A) Adenocarcinoma. Because this patient is a nonsmoker,
    adenocarcinoma is the most likely diagnosis.
    Patients with a history of smoking are more likely to
    develop small cell carcinoma, squamous cell carcinoma,
    and large cell carcinoma. It is not uncommon
    for patients to present with bone pain from metastasis
    as their initial manifestation of lung cancer.
138
Q
  1. A 60-year-old woman presents with a 6-month history
    of bilateral hip pain. Pain is more severe in the
    right hip than in the left hip. Patrick’s maneuver is
    markedly positive on the right side. On range of
    motion testing, the patient also complains of pain,
    and crepitus is noted in the right hip. A plain radiograph
    is ordered (Figure 5). What is this patient’s
    most likely diagnosis?
    (A) AVN
    (B) Hip fracture
    (C) Osteoarthritis
    (D) Trochanteric bursitis

Picture: hp_aug07_rqhip (1)

A
  1. (C) Osteoarthritis. Crepitus is the hallmark finding
    of osteoarthritis. Patrick’s maneuver confirms an anterior
    cause of the hip pain. This patient developed
    symptoms over 6 months, and AVN would likely have
    a more rapid course. Without a history of trauma,
    hip fracture is unlikely in this patient. Trochanteric
    bursitis causes lateral hip pain, not anterior hip pain.
139
Q
  1. A 25-year-old distance runner presents with lateral
    hip pain with every step. The patient is unable to
    jog. Ober’s test is positive. What is this patient’s
    most likely diagnosis?
    (A) Iliotibial band syndrome
    (B) Meralgia paresthetica
    (C) Sciatica
    (D) Trochanteric bursitis
A
  1. (A) Iliotibial band syndrome. Ober’s test is performed
    by having the patient lie on the unaffected
    side, and the affected leg is then abducted with the
    knee flexed to 90 degrees and released. Ober’s test
    is considered positive for iliotibial band syndrome if
    the thigh remains abducted when it is released. Sciatica
    causes posterior hip pain and is most likely due
    to a herniated disk in the lumbar spine. Meralgia
    paresthetica causes anterolateral thigh numbness
    and pain and is due to an entrapped lateral femoral
    cutaneous nerve. Trochanteric bursitis also causes
    lateral hip pain but is diagnosed by simple palpation
    of the lateral hip over the greater trochanter.
140
Q
  1. A 40-year-old obese man presents with numbness
    and tingling over the anterolateral right thigh. He
    reports a 20-lb weight gain over the past year. When
    asked about his clothing, he states that he has not
    purchased any new clothing. What is this patient’s
    most likely diagnosis?
    (A) Iliotibial band syndrome
    (B) Meralgia paresthetica
    (C) Sciatica
    (D) Trochanteric bursitis
A
  1. (B) Meralgia paresthetica. Meralgia paresthetica
    causes numbness and pain in the anterolateral
    thigh as a result of an entrapped lateral femoral cutaneous
    nerve. Risk factors for meralgia paresthetica
    include obesity, pregnancy, and ascites. In this case,
    the patient’s weight gain likely caused nerve entrapment.
    Upon further history, the patient reported
    that he was still wearing the same size of clothing;
    nerve entrapment may be due to wearing clothes
    that are too tight. Trochanteric bursitis and iliotibial
    band syndrome cause lateral hip pain as a result of
    overuse rather than compression. Sciatica is unlikely
    because it causes posterior hip pain.
141
Q
788. Tachyphylaxis to local anesthetics is most closely related to which of the following?
A. Speed of injection
B. Dosing interval
C. Temperature of local anesthetic
D. Volume of local anesthetic
E. pH of solution
A

788.
(B)
Tachyphylaxis is a well-known phenomenon associated with repeated injections of local anesthetics leading to
decreased effectiveness. Interestingly, the dosing interval seems most important in the development of
tachyphylaxis. If the dosing interval is short (and no pain between injections) tachyphylaxis does not develop.
However, with longer dosing intervals (and pain between injections) tachyphylaxis develops

142
Q
789. Which of the following techniques is LEAST effective in a treatment of pruritus from administration of
neuraxial opiates?
A. Nalbuphine 5 mg IV
B. Dexmedetomidine 30 μg IV
C. Diphenhydramine 50 mg IV
D. Hydroxyzine 20 mg IM
E. Propofol 10 mg IV
A

789.
(B)
The treatment of pruritus, the most common side effect of neuraxial opiates, is primarily with opioid
antagonists, mixed opioid agonist-antagonist, and antihistamine drugs (by their sedating effects). Nalbuphine
is a mixed opioid agonist-antagonist; diphenhydramine and hydroxyzine have antihistamine properties.
Propofol at very low doses (e.g., 10 mg) has been useful to treat pruritus not only induced by neuraxial opiates
but also the pruritus associated with cholestatic liver disease. Propofol does not affect analgesia, whereas
opioid antagonists and mixed agonist-antagonist may reverse some or all of the analgesia, depending upon
dose. Dexmedetomidine is a highly selective α2-receptor agonist that has a faster onset and shorter duration
of action compared with clonidine. Dexmedetomidine has analgesic properties, can potentiate neuraxial
analgesia when injected spinally, and can perhaps decrease the incidence of pruritus by reducing the narcotic
dose is used. It does not treat pruritus

143
Q
  1. The maximum dose of lidocaine containing 1:200,000 epinephrine that can be administered to a 70-kg patient
    for regional anesthesia (other than spinal anesthesia) is
    A. 50 mg
    B. 100 mg
    C. 200 mg
    D. 500 mg
    E. 1000 mg
A

790.
(D)
The maximum dose of local anesthetics containing 1:200,000 epinephrine that can be used for major nerve
blocks is lidocaine, 500 mg; mepivacaine, 500 mg; prilocaine, 600 mg; bupivacaine, 225 mg; etidocaine, 400
mg; and tetracaine, 200 mg

144
Q
791. Which of the following concentrations of epinephrine corresponds to a 1:200,000 mixture?
A. 0.5 μg/mL
B. 5 μg/mL
C. 50 μg/mL
D. 0.5 mg/mL
E. None of the above
A

791.
(B)
1:200,000 means 1 g = 1000 mg = 1,000,000 μg per 200,000 mL
1,000,000 μg/200,000 mL = 5 μg/mL

145
Q
  1. An anesthesia pain service consult is sought for a 78-year-old patient with a complaint of pain in the
    distribution of the trigeminal nerve. The patient has no other medical problems except a history of congestive
    heart failure for which he takes digoxin and thiazide. In addition to his chief complaint, the patient over the
    last 72 hours has complained of dysesthesia in the feet, difficulty with vision, and emesis times three. The
    most appropriate step at this time would be
    A. Trigeminal nerve block with bupivacaine
    B. Obtain neurologic workup for multiple sclerosis
    C. Administration of fentanyl and ondansetron
    D. Initiate therapy with carbamazepine
    E. Obtain a digoxin level
A

792.
(E)
The early signs of digitalis toxicity include loss of appetite and nausea and vomiting. In some patients there
may be pain that is similar to trigeminal neuralgia. Pain or discomfort in the feet and pain and discomfort in
the extremities may be a feature of digitalis toxicity. Transient visual disturbances (e.g., amblyopia, scotomata)
have been reported in patients with digitalis toxicity. In this patient, it would be prudent to obtain a digoxin level as an early part of the workup for these complaints. He may also have true trigeminal neuralgia, and
workup for this condition can be undertaken after digitalis toxicity has been ruled out

146
Q
793. Which of the following is the earliest sign of lidocaine toxicity?
A. Shivering
B. Nystagmus
C. Lightheadedness and dizziness
D. Tonic-clonic seizures
E. Nausea and vomiting
A

793.
(C)
Toxic reactions to local anesthetics are usually due to intravascular or intrathecal injection or to an excessive
dosage. The initial symptoms of local anesthetic toxicity are lightheadedness and dizziness. Patients also may
note perioral numbness and tinnitus. Progressive central nervous system (CNS) excitatory effects include
visual and auditory disturbances, shivering, muscular twitching, and ultimately, generalized tonic-clonic
seizures. CNS depression can ensue, leading to respiratory depression or arrest

147
Q
794. An analgesic effect similar to the epidural administration of 10 mg of morphine could be achieved by which
dose of intrathecal morphine?
A. 0.1 mg
B. 1 mg
C. 5 mg
D. 10 mg
E. There is no correlation
A

794.
(B)
The site of action of spinally administered opiates is the substantia gelatinosa of the spinal cord. Epidural
administration is complicated by factors related to dural penetration, absorption in fat, and systemic uptake;
therefore, the quantity of intrathecally administered opioid required to achieve effective analgesia is typically
much smaller. The ratio of epidural to intrathecal dose of morphine is approximately 10:1. Morphine is typically
given in doses of 3 to 10 mg in the lumbar epidural space. Intrathecal morphine dosage is 0.2 to 1.0 mg.
Onset time for epidural administration is 30 to 60 minutes with a peak effect in 90 to 120 minutes. Onset time
for intrathecal administration is shorter than epidural administration. Duration of 12 to 24 hours of analgesic
effect can be expected by either route

148
Q
795. Which local anesthetic undergoes the LEAST hepatic clearance
A. Chloroprocaine
B. Bupivacaine
C. Etidocaine
D. Prilocaine
E. Lidocaine
A

795.
(A)
Commonly injected local anesthetics are divided chemically into two groups: the aminoesters (esters) and the
aminoamides (amides). The esters include procaine, chloroprocaine and tetracaine (all have one letter i in the
name). The amides are lidocaine, mepivacaine, prilocaine, bupivacaine, levobupivacaine, etidocaine and
ropivacaine (all have two i’s in the name). The esters undergo plasma clearance by cholinesterases and have
relatively short half-lives, whereas the amides undergo hepatic clearance and have longer half-lives

149
Q
  1. Which of the following is the most important disadvantage of interscalene brachial plexus block compared
    with other approaches?
    A. Not suitable for operations on the shoulder
    B. Large volumes of local anesthetics required
    C. Frequent sparing of the ulnar nerve
    D. Frequent sparing of the musculocutaneous nerve
    E. High incidence of pneumothorax
A

796.
(C)
The major disadvantage of the interscalene block for hand and forearm surgery is that blockade of the inferior
trunk (C8-T1) is often incomplete. Supplementation of the ulnar nerve often is required. The risk of
pneumothorax is quite low, but blockade of the ipsilateral phrenic nerve occurs in up to 100% of blocks. This
can cause respiratory compromise in patients with significant lung disease

150
Q
  1. A 68-year-old woman is to undergo foot surgery under spinal anesthesia. Which of the following statements
    concerning the immediate physiologic response to the surgical incision is true?
    A. The cardiovascular response to stress will be blocked, but the adrenergic response will not
    B. The adrenergic response to stress will be blocked, but the cardiovascular response will not
    C. Both the adrenergic and cardiovascular responses will be blocked
    D. Neither the adrenergic or cardiovascular response will be blocked
    E. The cardiovascular response will be blocked but the adrenergic response will be augmented
A

797.
(C)
Surgical trauma includes a wide variety of physiologic responses. General anesthesia has no or only a slight
inhibitory effect on endocrine and metabolic responses to surgery. Regional anesthesia inhibits the nociceptive
signal from reaching the CNS and, therefore, has a significant inhibitory effect on the stress response,
including adrenergic, cardiovascular, metabolic, immunologic, and pituitary. This effect is most pronounced
with procedures on the lower part of the body and less with major abdominal and thoracic procedures. The
variable effect is probably due to unblocked afferents, i.e., vagal, phrenic, or sympathetic

151
Q
798. The “snap” felt just before entering the epidural space represents passage through which ligament?
A. Anterior longitudinal ligaments
B. Posterior longitudinal ligaments
C. Ligamentum flavum
D. Supraspinous ligament
E. Interspinous ligament
A

798.
(C)
The structures that are traversed by a needle placed in the midline prior to the epidural space are as follows:
skin, subcutaneous tissue, supraspinous ligament, interspinous ligament, and ligamentum flavum. The
ligamentum flavum is tough and dense and a change in the resistance to advancing the needle is often
perceived and to many feels like a “snap.” The anterior and posterior longitudinal ligaments bind the vertebral
bodies together. See also explanation and diagram in question 870

152
Q
799. The common element thought to be present in every case of cauda equina syndrome after continuous spinal
anesthesia is
A. Use of microcatheter
B. Maldistribution of local anesthetic
C. Administration of lidocaine
D. Addition of epinephrine
E. Hyperbaricity
A

799.
(B)
The symptoms of cauda equina syndrome include low back pain, bilateral lower extremity weakness, saddle
anesthesia and loss of bowel and bladder control. Pooling of local anesthetics in dependent areas of the spine
within the subarachnoid space has been identified as the causative factor in cases of cauda equina syndrome.
Microlumen catheters may enhance the nonuniform distribution of solutions within the intrathecal space, but
cauda equina syndrome has been associated with the use of larger catheters, 5% lidocaine with dextrose, and
2% lidocaine, as well as 0.5% tetracaine

153
Q
  1. A sciatic nerve block is performed in a healthy 26-year-old male patient for bunion surgery. Fifteen mL of
    1.5% mepivacaine is slowly injected after the landmarks are identified and a paresthesia is elicited in the
    great toe. In what order would the following nerve fibers be blocked?
    A. Sympathetic, proprioception, pain, motor
    B. Sympathetic, pain, proprioception, motor
    C. Motor, pain, proprioception, sympathetic
    D. Pain, proprioception, sympathetic, motor
    E. Pain, proprioception, motor, sympathetic
A

800.
(B)
Differential nerve blockade is a complex process where anatomic and chemical factors determine the
susceptibility of fibers to blockade by local anesthetics. Diameter, myelinization, and location within the nerve
trunk affect the onset and regression time. In general, the small unmyelinated sympathetic fibers are blocked
first, followed by unmyelinated C fibers (pain and temp), then small myelinated fibers (proprioception, touch,
pressure), and finally the large myelinated fibers (motor)

154
Q
  1. A 95-year-old woman has persistent and prolonged thoracic pain after a herpes zoster infection. Which of the
    treatments below would be the LEAST efficacious in the treatment of her pain?
    A. Oral amitriptyline
    B. Oral clonidine
    C. Topical capsaicin ointment
    D. Transcutaneous electrical nerve stimulation (TENS)
    E. Topical lidocaine patch
A
  1. B
    Acute herpes zoster is due to the reactivation of the varicella-zoster virus. Acute treatment includes
    symptomatic pain treatment and antiviral drugs (e.g., acyclovir). It is typically a benign and self-limiting disease
    in patients younger than 50 years of age. As one gets older, the incidence of postherpetic neuralgia (PHN)
    defined as pain persisting beyond the healing of the herpes zoster lesions increases. The incidence of PHN is
    about 50% in patients older than 50 years. Treatment of established PHN has been shown to be resistant to
    interventions and, thus, can be difficult. However, proven therapies include tricyclic antidepressants,
    anticonvulsants, topical local anesthetics (e.g., 5% lidocaine patch), topical capsaicin and TENS. Sympathetic
    blocks can provide excellent analgesia but are most useful during the more acute stages of the disease rather
    than during the late chronic stages. Sympathetic blocks in the acute stages may decrease the incidence of
    PHN. Oral clonidine, which is used to treat hypertension and opioid withdrawal, has not been shown to be an
    effective treatment for postherpetic neuralgia
155
Q
  1. The deep peroneal nerve innervates the
    A. Lateral aspect of the dorsum of the foot
    B. Entire dorsum of the foot
    C. Web space between the great toe and the second toe
    D. Web space between the third and fourth toes
    E. Medial aspect of the dorsum of the foot
A

802.
(C)
The deep peroneal nerve innervates the short extensors of the toes and the skin of the web space between
the great and second toe. The deep peroneal nerve is blocked at the ankle by infiltration between the tendons
of the anterior tibial and extensor hallucis longus muscle

156
Q
803. The correct arrangement of local anesthetics in order of their ability to produce cardiotoxicity from most to
least is
A. Bupivacaine, lidocaine, ropivacaine
B. Bupivacaine, ropivacaine, lidocaine
C. Lidocaine, bupivacaine, ropivacaine
D. Ropivacaine, bupivacaine, lidocaine
E. Lidocaine, ropivacaine, bupivacaine
A

803.
(B)
Central nervous system (CNS) toxicity from local anesthetics generally parallels anesthetic potency (e.g.,
bupivacaine is four times as potent as lidocaine, ropivacaine is three times as potent as lidocaine).
Cardiovascular (CV) toxicity occurs at a higher blood level than CNS toxicity. For bupivacaine and ropivacaine,
CV toxicity occurs at two times the CNS dose, whereas for lidocaine the CV toxicity occurs at seven times the
CNS toxicity levels, making lidocaine the least cardiotoxic, and bupivacaine the most cardiotoxic of the listed
local anesthetics

157
Q
  1. Allodynia is defined as
    A. Spontaneous pain in an area or region that is anesthetic
    B. Pain initiated or caused by a primary lesion or dysfunction in the nervous system
    C. An unpleasant abnormal sensation, whether spontaneous or evoked
    D. An increased response to a stimulus that is normally painful
    E. Pain caused by a stimulus that does not normally provoke pain
A

804.
(E)
The International Association for the Study of Pain (IASP) has defined several pain terms. Anesthesia
dolorosa refers to spontaneous pain in an area or region that is anesthetic. Neuropathic pain is pain initiated
or caused by a primary lesion or dysfunction in the nervous system. Dysesthesia is an unpleasant abnormal
sensation, whether spontaneous or evoked. Hyperalgesia is an increased response to a stimulus that is
normally painful. Allodynia is pain caused by a stimulus that does not normally provoke pain

158
Q
805. The primary mechanism by which the action of tetracaine is terminated when used for spinal anesthesia is
A. Systemic absorption
B. Uptake into neurons
C. Hydrolysis by pseudocholinesterase
D. Hydrolysis by nonspecific esterases
E. Spontaneous degradation at 37° C
A

805.
(A)
Ester local anesthetics are hydrolyzed by cholinesterase enzymes that are present mainly in plasma and, in a
smaller amount, in the liver. Because there are no cholinesterase enzymes present in cerebrospinal fluid
(CSF), the anesthetic effect of tetracaine will persist until it is absorbed into systemic circulation. The rate of
hydrolysis varies, with chloroprocaine being fastest, procaine intermediate, and tetracaine the slowest. Toxicity
is inversely related to the rate of hydrolysis; tetracaine is, therefore, the most toxic

159
Q
  1. Complex regional pain syndrome type I (reflex sympathetic dystrophy) is differentiated from complex regional
    pain syndrome type II (causalgia) by knowledge of its
    A. Etiology
    B. Chronicity
    C. Affected body region
    D. Type of symptoms
    E. Rapidity of onset
A

806.
(A)
Complex regional pain syndrome type I or CRPS type I also called reflex sympathetic dystrophy (RSD) is a
clinical syndrome of continuous burning pain, usually occurring after minor trauma. Patients present with
variable sensory, motor, autonomic, and trophic changes. Complex regional pain syndrome type II or CRPS
type II (causalgia) exhibits the same features of reflex sympathetic dystrophy, but the etiology is usually major
traumatic damage to large nerves (e.g., median nerve of the upper extremity or tibial division of the sciatic
nerve in the lower extremity)

160
Q
807. The primary determinant of local anesthetic potency is
A. pKa
B. Molecular weight
C. Lipid solubility
D. Concentration
E. Protein binding
A

807.
(C)
The potency of local anesthetics is directly related to their lipid solubility. In general, the speed or onset of
action of local anesthetics is related to the pKa of the drug. Drugs with lower pKa values have a higher
amount of non-ionized molecules at physiologic pH and penetrate the lipid portion of nerves faster (an
exception is chloroprocaine, which has a fast onset of action that may be related to the higher concentration of
drug used). The amount of protein binding seems related to the duration of action of local anesthetics (more
protein binding has longer duration of action)

161
Q
  1. Which of the following would have the greatest effect on the level of sensory blockade after a subarachnoid
    injection of hyperbaric 0.75% bupivacaine?
    A. Coughing during placement of the block
    B. Addition of epinephrine to the local anesthetic solution
    C. Barbotage
    D. Patient weight
    E. Patient position
A

808.
(E)
Many factors have an effect on the sensory level after a subarachnoid injection. The baricity of the solution
and the patient position are the most important determinants of sensory level. The other listed options have
little to no effect on sensory level. Patient height also has little effect on sensory level

162
Q
  1. Which of the following local anesthetics would produce the lowest concentration in the fetus relative to the
    maternal serum concentration during a continuous lumbar epidural?
    A. Etidocaine
    B. Bupivacaine
    C. Lidocaine
    D. Chloroprocaine
    E. Mepivacaine
A

809.
(D)
Chloroprocaine is an ester local anesthetic that is rapidly metabolized by pseudocholinesterase. With the
epidural injection of chloroprocaine, very little drug is available to cross the placenta, because the half-life is
about 45 seconds (and that which crosses is also rapidly metabolized making fetal effects essentially non-significant). The amide local anesthetics undergo liver metabolism and have relatively long half-lives, but with
prolonged epidural administration may accumulate in the fetus

163
Q
  1. Severe hypotension associated with high spinal anesthesia is caused primarily by
    A. Decreased cardiac output secondary to decreased preload
    B. Decreased systemic vascular resistance
    C. Decreased cardiac output secondary to bradycardia
    D. Decreased cardiac output secondary to decreased myocardial contractility
    E. Increased shunting through metarterioles
A

810.
(A)
Hypotension with a high spinal anesthesia is related to sympathetic blockade; venodilation (decreases
preload), arterial dilation (decreases afterload) and a decrease in heart rate (cardioaccelerator fibers T1-T4
blockade and a fall in right atrial filling that affects the intrinsic chronotropic stretch receptors). With a high
spinal, the decrease in venous dilation is the predominant cause of hypotension

164
Q
  1. Select the one true statement concerning phantom limb pain.
    A. Most phantom limb pain becomes more severe with time
    B. Most amputees do not experience phantom limb pain
    C. Nerve blocks may be used to treat phantom limb pain
    D. Trauma amputees have a higher incidence of phantom limb pain than nontrauma amputees
    E. The incidence of phantom limb pain increases with more distal amputations
A

811.
(C)
The incidence of phantom limb pain is estimated to be 60% to 85%. The incidence of phantom limb pain does
not differ between traumatic and nontraumatic amputees. The incidence of phantom pain increases with more
proximal amputation. Although very difficult to treat, nerve blocks are commonly used in an attempt to treat
phantom pain. These include trigger point injections, peripheral and central nerve blocks, and sympathetic
blocks

165
Q
812. Which of the following local anesthetics used for intravenous regional anesthesia (Bier block) is most rapidly
metabolized and thus, least toxic?
A. Lidocaine
B. Ropivacaine
C. Mepivacaine
D. Prilocaine
E. Etidocaine
A

812.
(D)
Prilocaine is the most rapidly metabolized of the amide local anesthetics and therefore least toxic. 2-
Chloroprocaine is hydrolyzed rapidly in the blood and, therefore, would appear to be ideal, but it has been
associated with a high incidence of thrombophlebitis and is therefore not recommended. To avoid toxicity,
maximum doses are as follows: prilocaine, 3 to 4 mg/kg; lidocaine, 1.5 to 3 mg/kg; ropivacaine, 1.2 to 1.8
mg/kg. Bupivacaine is not recommended for Bier blocks because of reports of cardiovascular toxicity and
death that have occurred

166
Q
  1. Select the FALSE statement regarding spinal anatomy and spinal anesthesia.
    A. The addition of phenylephrine to lidocaine will prolong spinal anesthesia
    B. A high thoracic sensory block will result in total sympathetic blockade
    C. The largest vertebral interspace is L5-S1
    D. The dural sac extends to the S4-S5 interspace
    E. Tetracaine provides longer anesthesia than does procaine
A

813.
(D)
Both phenylephrine and epinephrine will prolong a spinal anesthetic when administering lidocaine. The Taylor
approach for spinal anesthesia uses a paramedian approach to the L5-S1 interspace—the largest interspace
of the vertebral column. The sympathetic nervous system originates in the thoracic and lumbar spinal cord T1-
L3; therefore, a high thoracic sensory level can cause a complete sympathetic block. The dural sac extends to
S2-S3, not S4-S5. The spinal cord extends to L3 in the infant and L1-L2 in adults

167
Q
  1. Four days after a left total hip arthroplasty, an obese 62-year-old woman complains of severe back pain in
    the region where the epidural was placed. Over the ensuing 72 hours, the back pain gradually worsens and a
    severe aching pain that radiates down the left leg to the knee develops. The most likely diagnosis is
    A. Epidural abscess
    B. Epidural hematoma
    C. Anterior spinal artery syndrome
    D. Arachnoiditis
    E. Meralgia paresthetica
A

814.
(A)
Development of an epidural abscess is fortunately an exceedingly rare complication of spinal and epidural
anesthesia. Most anesthetic related epidural abscesses are associated with epidural catheters. When an
epidural abscess is developing, prompt recognition and treatment are essential if permanent sequelae are to
be avoided. Symptoms from an epidural abscess may not become apparent until several days (mean 5 days)
after placement of the block. There are four clinical stages of epidural abscess symptom progression. Initially,
localized back pain develops. Second stage includes nerve root or radicular pain. The third stage involves
motor and sensory deficits followed by the last stage of paraplegia. Unlike an epidural hematoma, in which
severe back pain is the key feature, patients with epidural abscesses will complain of radicular pain
approximately 3 days after development of the back pain. Anterior spinal artery syndrome is characterized
predominantly by motor weakness or paralysis of the lower extremities. Meralgia paresthetica is related to
entrapment of the lateral femoral cutaneous nerve as it courses below the inguinal ligament and is associated
with burning pain over the lateral aspect of the thigh. It is not a complication of epidural anesthesia

168
Q
815. Which of the following choices is NOT consistent with a limb affected by complex regional pain syndrome?
A. Osteoporosis
B. Allodynia
C. Dermatomal distribution of pain
D. Atrophy of the involved extremity
E. Hyperesthesia
A

815.
(C)
Complex regional pain syndromes are associated with trauma. The main feature is burning and continuous
pain that is exacerbated by normal movement, cutaneous stimulation, or stress, usually weeks after the injury.
The pain is not anatomically distributed. Other associated features include cool, red, clammy skin and hair
loss in the involved extremity. Chronic cases may be associated with atrophy and osteoporosis

169
Q
  1. The main advantage of neurolytic nerve blockade with phenol versus alcohol is
    A. Denser blockade
    B. Blockade is permanent
    C. The effects of the block can be evaluated immediately
    D. The block is less painful
    E. Phenol is selective for sympathetic fibers
A

816.
(D)
Neurolytic blockade with phenol (6% to 10% in glycerine) is painless because phenol has a dual action as
both a local anesthetic and a neurolytic agent. The initial block wears off over a 24-hour period, during which
time neurolysis occurs. For this reason you must wait a day to determine effectiveness of the neurolytic block.
Alcohol (100% ethanol) is painful on injection and should be preceded by local anesthetic injection.
Unfortunately, there is no neurolytic agent that affects only sympathetic fibers

170
Q
  1. How much local anesthetic should be administered per spinal segment to patients between 20 and 40 years
    of age receiving a lumbar epidural anesthetic?
    A. 0.25 to 0.5 mL
    B. 0.5 to 1.0 mL
    C. 1 to 2 mL
    D. 2 to 3 mL
    E. 3 to 5 mL
A

817.
(C)
In general, each 1-2 mL of local anesthetic will anesthetize about one spinal segment in the 20 to 40-year-old
patient. Because of the negative intrathoracic pressure transmitted to the epidural space with breathing, about
two thirds of the segments are blocked above the level of the lumbar placement and one third of segments are
blocked below the injection. For example, to achieve a T4 block when an epidural is placed at the L2-L3
space about 10 segments above and 5 segments below the epidural would be needed (15 segments) or about
20-25 mL. As one gets older, the dose of local anesthetic mL/segment decreases (e.g., 80 year old may need 0.75-1.5 mL/segment). Also, pregnant patients are more sensitive to local anesthetics and reduced doses are
needed

171
Q
818. The artery of Adamkiewicz most frequently arises from the aorta at which spinal level?
A. T1-T4
B. T5-T8
C. T9-T12
D. L1-L4
E. L5-S3
A

818.
(C) The artery of Adamkiewicz is also called the arteria radicularis magna and is one of the “feeder” arteries for
the anterior spinal artery. Damage to this artery can lead to ischemia in the thoracolumbar region and may
result in paraplegia. The origin of this artery is variable

172
Q
  1. The anterior and posterior spinal arteries originate from the
    A. Common carotid and vertebral arteries, respectively
    B. Internal carotid and vertebral arteries, respectively
    C. Internal carotid and posterior cerebral arteries, respectively
    D. Vertebral and anterior cerebellar arteries, respectively
    E. Vertebral, radicular arteries and the posterior inferior cerebellar arteries, respectively
A

819.
(E)
The one anterior spinal artery supplies about 75% of the blood flow to the spinal cord (motor tracts) and
arises from the vertebral arteries and radicular arteries from the aorta. It descends in front of the anterior
longitudinal sulcus of the spinal cord. The two posterior spinal arteries supply about 25% of the blood flow to
the spinal cord (sensory tracts) and arise from the posterior and inferior cerebellar arteries, the vertebral
arteries, and the radicular arteries

173
Q
  1. Important landmarks for performing a sciatic nerve block (classic approach of Labat) include
    A. Iliac crest, sacral hiatus, greater trochanter
    B. Iliac crest, coccyx, and greater trochanter
    C. Posterior superior iliac spine, coccyx, and greater trochanter
    D. Posterior superior iliac spine, greater trochanter and sacral hiatus
    E. Posterior superior iliac spine and greater trochanter
A

820.
(D)
To perform a sciatic nerve block, first draw a line from the posterior superior iliac spine to the greater
trochanter, then draw a 5-cm line perpendicular from the midpoint of this line caudally and a second line from
the sacral hiatus to the greater trochanter. The intersection of the second line with the perpendicular line marks the point of entry

174
Q
  1. A 36-year-old female patient is undergoing thyroidectomy under a deep cervical plexus nerve block. Which of
    the following complications would be LEAST likely with this block?
    A. Horner’s syndrome
    B. Subarachnoid injection
    C. Blockade of the recurrent laryngeal nerve
    D. Blockade of the spinal accessory nerve
    E. Vertebral artery injection
A

821.
(D)
Complications of deep cervical plexus block include injection of the local anesthetic into the vertebral artery,
subarachnoid space, or epidural space. Other nerves that may be anesthetized include the phrenic nerve
(which is why bilateral deep cervical plexus blocks should be performed with caution, if at all), and the
recurrent laryngeal nerve

175
Q
  1. A retrobulbar block anesthetizes each of the following nerves EXCEPT
    A. Ciliary nerves
    B. Cranial nerve IV (trochlear nerve)
    C. Cranial nerve III (oculomotor nerve)
    D. Cranial nerve VI (abducens nerve)
    E. Maxillary branch of the trigeminal nerve
A

822.
(E)
A retrobulbar block anesthetizes the three cranial nerves responsible for movement of the eye. The ciliary
nerves are also blocked, providing anesthesia to the conjunctiva, cornea, and uvea. The ophthalmic branch of
the trigeminal nerve provides sensory innervation to the skin of the forehead, cornea, and eyelid. This branch
of the trigeminal nerve may be blocked, but the maxillary branch would be spared

176
Q
823. Which of the following muscles of the larynx is innervated by the external branch of the superior laryngeal
nerve?
A. Vocalis muscle
B. Thyroarytenoid muscles
C. Posterior cricoarytenoid muscle
D. Oblique arytenoid muscles
E. Cricothyroid muscle
A

823.
(E)
The vagus nerve innervates the airway by two branches: the superior laryngeal nerves and the recurrent
laryngeal nerves. All the muscles of the larynx are innervated by the recurrent laryngeal nerve except for the
cricothyroid muscle. The superior laryngeal nerve divides into the internal and external laryngeal branches.
The external laryngeal branch innervates the cricothyroid muscle. The internal laryngeal branch provides
sensory fibers to the cords, epiglottis and the arytenoids

177
Q
824. All the following agents are acceptable for use in a Bier block EXCEPT
A. 0.5% Lidocaine
B. 0.5% Mepivacaine
C. 0.5% Procaine
D. 0.5% Prilocaine
E. 0.25% Bupivacaine
A

824.
(E)
Because of the potential for cardiotoxicity and because bupivacaine has no advantages over other local
anesthetics in this setting, it is no longer recommended for use in intravenous regional anesthesia

178
Q
825. The stellate ganglion lies in closest proximity to which of the following vascular structures?
A. Common carotid artery
B. Internal carotid artery
C. Vertebral artery
D. Axillary artery
E. Aorta
A

825.
(C)The stellate ganglion usually lies in front of the neck of the first rib. The vertebral artery lies anterior to the
ganglion as it has just originated from the subclavian artery. After passing over the ganglion, it enters the
vertebral foramen and lies posterior to the anterior tubercle of C6

179
Q
826. Which of the following structures in the antecubital fossa is the most medial?
A. Brachial artery
B. Radial nerve
C. Tendon of the biceps
D. Median nerve
E. Musculocutaneous nerve
A

826.
(D)
The median nerve is the most medial structure in the antecubital fossa. To block this nerve, first the brachial
artery is palpated at the level of the intercondylar line between the medial and lateral epicondyles, and then a
needle is inserted just medial to the artery and directed perpendicularly to the skin

180
Q
  1. During placement of an epidural in a 78-year-old patient scheduled for a total knee arthroplasty, the patient
    complains of a sharp sustained pain radiating down his left leg as the catheter is inserted to 2 cm. The most
    appropriate action at this time would be
    A. Leave the catheter at 2 cm, give test dose
    B. Give small dose to relieve pain then advance 1 cm
    C. Withdraw the catheter 1 cm, give test dose
    D. Withdraw needle and catheter, reinsert in a new position
    E. Abandon epidural technique, place long-acting spinal anesthetic
A

827.
(D)
When an epidural catheter is placed without fluoroscopic guidance, the exact location of the needle tip relative
to the anatomic structures of the back can only be surmised. If malposition of either the needle or the catheter
is suspected, it is prudent to withdraw the entire apparatus and reinsert a second time. In this case, it is
possible that the catheter tip has found its way into a nerve root. Under these circumstances, injection of a
local anesthetic or narcotic could produce pressure that would lead to ischemia and possible neurologic
damage. During placement or injection of an epidural catheter, a paresthesia is always a warning sign that should be heeded

181
Q
828. Cutaneous innervation of the plantar surface of the foot is provided by the
A. Sural nerve
B. Posterior tibial nerve
C. Saphenous nerve
D. Deep peroneal nerve
E. Superficial peroneal nerve
A

828.
(B)
There are five nerves that supply the ankle and foot: the posterior tibial, sural, superficial and deep peroneal,
and saphenous nerves. These nerves are superficial at the level of the ankle and are easy to block. The
posterior branch of the tibial nerve gives rise to the medial and lateral plantar nerves, which supply the plantar
surface of the foot

182
Q
  1. Which of the following local anesthetics has the lowest ratio of dosage required for cardiovascular collapse to
    dosage required for central nervous system toxicity?
    A. Lidocaine
    B. Etidocaine
    C. Bupivacaine
    D. Prilocaine
    E. Chloroprocaine
A

829.
(C)
In general, in both in vivo and in vitro studies there is an overall direct correlation between anesthetic’s
potency and its direct depressant effect on myocardial contractility. The ratio of dosage required for
cardiovascular collapse in animal models compared with that required to produce neurologic symptoms is the
lowest for bupivacaine, levo-bupivacaine and ropivacaine (2.0). Ratios for other local anesthetics are as
follows: prilocaine, 3.1; procaine and chloroprocaine, 3.7; etidocaine, 4.4; lidocaine and mepivacaine, 7.1

183
Q
  1. A 57-year-old patient is scheduled for hemorrhoidectomy. The patient has a history of mild chronic
    obstructive pulmonary disease, hypertension, and traumatic foot amputation from a tractor accident. His only
    hospitalizations were for two suicide attempts related to phantom limb sensations 10 years ago. He takes
    phenelzine (Nardil), thiazide, and potassium. Which of the following anesthetic techniques would be most
    appropriate for this patient?
    A. Spinal anesthetic with 0.5% hyperbaric bupivacaine
    B. Epidural anesthetic with 0.5% bupivacaine
    C. Local infiltration with lidocaine and epinephrine, sedation with propofol and meperidine
    D. General anesthesia with thiopental sodium (Pentothal), succinylcholine, nitrous oxide, isoflurane,
    meperidine
    E. General anesthesia with propofol, succinylcholine, nitrous oxide, fentanyl
A

830.
(E)
Reactivation of phantom limb sensations has been reported in patients who have received both spinal and
epidural anesthetics (90% in some series). In the majority of these cases (80%), phantom limb sensation
persisted until the block receded. With a history of a phantom limb sensations that drove this patient to
attempt suicide, it is probably wise to avoid spinal and epidural anesthetics. Phenelzine (Nardil) is a
monoamine oxidase (MAO) inhibitor that is occasionally used for the treatment of depression. Any anesthetic
or combination of techniques that involves meperidine is contraindicated in patients receiving MAO inhibitors.
The combination of meperidine and MAO inhibitors has been associated with hyperthermia, hypotension,
hypertension, ventilatory depression, skeletal muscle rigidity, seizures, and coma. Because of this unfavorable
drug interaction, meperidine should be avoided in patients receiving MAO inhibitors. Accordingly, the only
acceptable choice in this question would be general anesthesia with propofol, succinylcholine, nitrous oxide,
and fentanyl. As an interesting side point, the drug phenelzine prolongs the duration of action of
succinylcholine by decreasing plasma cholinesterase activity

184
Q
  1. If the recurrent laryngeal nerve were transected bilaterally, the vocal cords would
    A. Be in the open position
    B. Be in the closed position
    C. Be in the intermediate position (i.e., 2-3 mm apart)
    D. Not be affected unless the superior laryngeal nerve were also injured
    E. Appear exactly the same as if an intubating dose of succinylcholine were given
A

831.
(C)
The recurrent laryngeal nerve innervates all the muscles of the larynx except the cricothyroid muscle, which
tenses the vocal cords and is innervated by the external branch of the superior laryngeal nerve. With bilateral
transections of the recurrent laryngeal nerve, the vocal cords lie within 2 to 3 mm of the midline. The airway
maybe inadequate and a tracheostomy may be needed

185
Q
832. A 63-year-old woman undergoes total knee arthroplasty under spinal anesthesia. Two days later she
complains of a severe headache on the left side of her head. Pain intensity is not related to posture. The
LEAST likely cause of this headache is
A. Caffeine withdrawal
B. Malingering
C. Viral illness
D. Migraine
E. Postdural puncture headache
A

832.
(E)
Postdural puncture headache (PDPH) will have a postural component. When supine, the headache is usually
gone but may be mild in some cases. When the head is elevated the headache may be severe, is bilateral
and may be associated with diplopia, nausea and vomiting. The headache pain is typically frontal and/or
occipital in location

186
Q
833. What is the correct order of structures (from cephalad to caudad) in the intercostal space?
A. Nerve, artery, vein
B. Vein, nerve, artery
C. Vein, artery, nerve
D. Artery, nerve, vein
E. Artery, vein, nerve
A

833.
(C)
VAN (Vein, Artery, Nerve) describes the anatomical relationship of the intercostal structures deep to the lower
border of the ribs from cephalad to caudal direction. The block is performed by walking off the inferior edge of
the rib typically about 5 to 7 cm from midline. The two principle risks are pneumothorax and intravascular
injection of local anesthetics. Because of the close proximity of the vein and artery to the nerve, intercostal
blocks have relatively high blood levels as compared to other blocks (e.g., epidural, brachial plexus, infiltration)
and caution with dose is needed if many levels are blocked

187
Q
834. Which of the following types of regional anesthesia is associated with the greatest serum concentration of
lidocaine?
A. Intercostal
B. Caudal
C. Epidural
D. Brachial plexus
E. Femoral nerve block
A

834.
(A)
The site of injection of the local anesthetic is one of the most important factors influencing systemic local
anesthetic absorption and toxicity. The degree of absorption from the site of injection depends on the blood
supply to that site. Areas that have the greatest blood supply have the greatest systemic absorption. For this
reason, the greatest plasma concentration of local anesthetic occurs after an intercostal block, followed by
caudal epidural, lumbar epidural, brachial plexus, sciatic/femoral nerve block, and subcutaneous

188
Q
  1. Differences in which of the following local anesthetic properties account for the fact that the onset of an
    epidural block with 3% 2-chloroprocaine is more rapid than 2% lidocaine?
    A. Protein binding
    B. pKa
    C. Lipid solubility
    D. Concentration
    E. Ester versus amide structure
A

835.
(D)
Local anesthetics are weak bases. The neutral (non-ionized) form of the molecule is able to pass through the
lipid nerve cell membrane, whereas the ionized (protonated) form actually produces anesthesia.
Chloroprocaine has the highest pKa of local anesthetics, meaning that a greater percentage of it will exist in
the ionized form at any given pH than any of the other local anesthetics. Despite this fact, 3% chloroprocaine
has a more rapid onset than 2% lidocaine, presumably because of the greater number of molecules
(concentration). If one compares onset time for 1.5% lidocaine against 1.5% chloroprocaine, the former will
have a more rapid onset

189
Q
  1. A 69-year-old man with a history of diabetes mellitus and chronic renal failure is to undergo placement of a
    dialysis fistula under regional anesthesia. During needle manipulation for a supraclavicular brachial plexus
    block, the patient begins to cough and complain of chest pain and shortness of breath. The most likely
    diagnosis is
    A. Angina
    B. Pneumothorax
    C. Phrenic nerve irritation
    D. Intravascular injection of local anesthetic
    E. Intrathecal injection of local anesthetic
A
  1. (B)
    The risk of pneumothorax is a significant limitation for supraclavicular brachial plexus blocks (incidence 0.5%- 6% depending upon experience). Furthermore, the technique is difficult to teach and describe. For these
    reasons, this block should not be performed in patients in whom a pneumothorax or phrenic nerve block
    (40%-60% of patients) would result in significant dyspnea or respiratory distress. A pneumothorax should be
    considered if the patient begins to complain of chest pain or shortness of breath or begins to cough during
    placement of supraclavicular brachial plexus block
190
Q
837. Which of the following nerves is located immediately lateral to the trachea?
A. Vagus
B. Recurrent laryngeal
C. Phrenic
D. Long thoracic
E. Spinal accessory
A

837.
(B)
The structures in the neck from medial to lateral are the recurrent laryngeal nerve, carotid artery, vagus nerve,
internal jugular vein, and phrenic nerve

191
Q
838. If a needle is introduced 1.5 cm inferior and 1.5 cm lateral to the pubic tubercle, to which nerve will it lie in
close proximity?
A. Obturator nerve
B. Femoral nerve
C. Lateral femoral cutaneous nerve
D. Sciatic nerve
E. Ilioinguinal nerve
A

838.
(A)
The obturator nerve provides variable cutaneous innervation of the thigh. An obturator nerve block is achieved
by placement of the needle 1 to 2 cm lateral to and 1 to 2 cm below the pubic tubercle. After contact with the
pubic bone, the needle is withdrawn and walked cephalad to identify the obturator canal. Between 10 and 15
mL of local anesthetic should be placed in the canal. If a nerve stimulator is used, contraction of the adductor
muscles with nerve stimulation indicates proximity to the nerve

192
Q
  1. The most common complication associated with a supraclavicular brachial plexus block is
    A. Blockade of the phrenic nerve
    B. Intravascular injection into the vertebral artery
    C. Spinal blockade
    D. Blockade of the recurrent laryngeal nerve
    E. Pneumothorax
A

839.
(A)
The most serious complication associated with a supraclavicular brachial plexus block is pneumothorax. The
most common complication is a phrenic nerve block which is usually mild and relatively common (40%-60% of
blocks). Bilateral supraclavicular blocks however, are not recommended due to the possibility of bilateral
phrenic nerve paralysis or pneumothoraces. Other potential complications include Horner’s syndrome, nerve
damage or neuritis, or intravascular injection

193
Q
840. Which portion of the upper extremity is not innervated by the brachial plexus?
A. Posterior medial portion of the arm
B. Elbow
C. Lateral portion of the forearm
D. Medial portion of the forearm
E. Anterolateral portion of the arm
A

840.
(A)
The arm receives sensory innervation from the brachial plexus except for the shoulder, which is innervated by
the cervical plexus, and the posterior medial aspect of the arm, which is supplied by the intercostobrachial
nerve

194
Q
841. Which section of the brachial plexus is blocked with a supraclavicular block?
A. Roots
B. Trunks
C. Divisions
D. Cords
E. Branches
A

841.
(B)
The brachial plexus starts out at the root level from the ventral rami of C5-T1 with a small amount from C4
and T2. These roots at the level of the scalene muscle become the 3 trunks: superior, middle and inferior.
The trunks then divide into the dorsal and ventral divisions at the lateral edge of the first rib. When the
divisions enter the axilla, they become the cords: posterior, lateral and medial. At the lateral border of the
pectoralis muscle they become the five peripheral nerves: radial, musculocutaneous, median, ulnar and
axillary. The plexus is blocked at the distal level of the trunks just before they become divisions. Here a small
volume of anesthetic is required and no part of the plexus is spared, as with axillary or interscalene block. The
block can be performed with the arm in any position

195
Q
842. A celiac-plexus block would NOT effectively treat pain resulting from a malignancy involving which of the
following organs?
A. Uterus
B. Adrenal gland
C. Stomach
D. Pancreas
E. Gallbladder
A

842.
(A)
The celiac plexus innervates most of the abdominal viscera, including the lower esophagus, stomach, all of
the small intestine and the large intestine up to the splenic flexure as well as the pancreas, liver, biliary tract,
spleen, kidneys, adrenal glands and omentum. The pelvic organs (e.g., uterus, ovaries, prostate, distal colon)
are supplied by the hypogastric plexus

196
Q
  1. A healthy 27-year-old female stepped on a nail and is to undergo débridement of a wound on her right great
    toe. She is anxious about general anesthesia but agrees to an ankle block with mild sedation. Which nerves
    must be adequately blocked in order to perform the surgery?
    A. Deep peroneal, posterior tibial, saphenous, sural
    B. Deep peroneal, saphenous, superficial peroneal, sural
    C. Deep peroneal, posterior tibial, superficial peroneal, sural
    D. Deep peroneal, posterior tibial, saphenous, superficial peroneal
    E. Deep peroneal, posterior tibial, saphenous
A

843.
(D)
The great toe is innervated by the deep peroneal, posterior tibial, superficial peroneal, and occasionally the
saphenous nerve. All four of these nerves should be blocked for surgery on the great toe

197
Q
  1. A 54-year-old man is administered morphine via patient-controlled analgesia (PCA) pump after a left total hip
    arthroplasty. The pump is programmed to deliver a maximum dose of 2 mg every 15 minutes (lockout time)
    as needed for patient comfort. The total maximum dose that can be delivered in 4 hours is 30 mg. On the
    first day the patient receives 15 doses every 4 hours by pressing the delivery button every 15 to 18 minutes.
    How should his pain control be further managed?
    A. Discontinue the PCA pump and administer intramuscular morphine
    B. Increase the lockout time from 15 to 25 minutes
    C. Change the analgesic from morphine to fentanyl
    D. Increase the dose to 3 mg every 15 minutes as needed up to a total maximum dose of 40 mg every 4
    hours
    E. Make no changes
A

844.
(D)
Frequent dosing by a patient receiving postoperative analgesia through a PCA pump suggests the need to
increase the magnitude of the dose. It is important to keep in mind that a patient should be given a sufficient
loading dose of narcotic before initiative therapy with a PCA pump. Otherwise, the patient will be playing the
frustrating game of “catch up”

198
Q
  1. The mechanism of the TENS unit in relieving pain is
    A. Direct electrical inhibition of type A-delta and C fibers
    B. Depletion of neurotransmitter in nociceptors
    C. Hyperpolarization of spinothalamic tract neurons
    D. Activation of inhibitory neurons
    E. Distortion of nociceptors
A

845.
(D)
Transcutaneous electrical nerve stimulation (TENS) is low-intensity electrical stimuli (2 and 100 Hz,) that
produces a tingling or vibratory sensation. It is thought that TENS units produce analgesia by releasing
endogenous endorphins. These endorphins have an inhibitory effect at the spinal cord level and augment
descending inhibitory pathways

199
Q
846. Epidural use of which of the following opioids would result in the greatest incidence of delayed respiratory
depression?
A. Sufentanil
B. Fentanyl
C. Morphine sulfate
D. Hydromorphone
E. Methadone
A

846.
(C)
Although the more hydrophilic drugs such as morphine have a longer duration of action of analgesia, they also
have a higher potential for inducing delayed respiratory depression through cephalad migration in the CNS, as
compared with the more lipid-soluble drugs listed in this question

200
Q
  1. A 21-year-old patient reports tingling in her thumb during cesarean section under epidural anesthesia. To
    which dermatomal level would this correspond?
    A. C4
    B. C5
    C. C6
    D. C7
    E. C8
A

847.
(C)
The thumb corresponds to dermatome C6, the second and middle fingers correspond to dermatome C7, and
the fourth and little fingers correspond to dermatome C8

201
Q
  1. Which of the following would hasten the onset and increase the clinical duration of action of a local
    anesthetic, and provide the greatest depth of motor and sensory blockade when used for epidural
    anesthesia?
    A. Addition to 1:200,000 epinephrine
    B. Increasing the volume of local anesthetic
    C. Increasing the concentration of local anesthetic
    D. Increasing the dose
    E. Placing the patient in the head-down position
A

848.
(D)
Increasing the total dose (mass) of local anesthetic is more efficacious in hastening the onset and increasing
the duration of an epidural anesthetic than increasing the volume or increasing the concentration (while
holding the total dose constant)

202
Q
  1. Select the FALSE statement concerning neurolytic nerve blocks.
    A. There is little difference in the efficacy between alcohol and phenol
    B. Destruction of peripheral nerves can be followed by a denervation hypersensitivity that is worse than
    the original pain
    C. Neurolytic blocks should be reserved for patients with short life expectancies
    D. Neurolytic blockade with phenol is permanent
    E. Intrathecal neurolysis may be an effective management for certain pain conditions
A

849.
(D)
Alcohol and phenol are similar in their ability to cause nonselective damage to neural tissues. Alcohol causes
pain when injected and sometimes is mixed with bupivacaine, whereas phenol is relatively painless. Neural
tissue will regenerate; therefore, neurolytic blocks are never “permanent” and neurolysis can lead to a
denervation hypersensitivity, which can be extremely painful

203
Q
  1. The addition of epinephrine to epidural bupivacaine will
    A. Prolong motor blockade only
    B. Prolong sensory blockade only
    C. Prolong motor and sensory blockade
    D. Shorten duration of sensory blockade
    E. Have no effect on either duration of motor or sensory blockade
A

850.
(B)
Epinephrine’s effect on the duration of anesthesia depends on the local anesthetic and the site. Infiltration and
peripheral block duration with most agents will be prolonged with epinephrine. The addition of epinephrine to
epidural 0.5% or 0.75% bupivacaine has not been shown to increase the duration of the motor blockade but
does extend the duration of the sensory block. The effect of epinephrine is greater for the intermediate
duration local anesthetics lidocaine and mepivacaine

204
Q
  1. The epidural administration of a mixture of chloroprocaine and bupivacaine would have
    A. A latency similar to chloroprocaine with a duration of action similar to bupivacaine
    B. A latency shorter than chloroprocaine with a duration of action longer than bupivacaine
    C. A latency shorter than chloroprocaine with a duration of action similar to bupivacaine
    D. A latency longer than chloroprocaine with a duration of action similar to chloroprocaine
    E. A latency longer than chloroprocaine with a duration of action shorter than bupivacaine
A

851.
(E)
Mixtures of local anesthetics have been used to take advantage of the short latency of certain agents
(chloroprocaine) and the long duration of other agents (bupivacaine). Duration of epidural anesthesia by
mixtures of chloroprocaine and bupivacaine has been shown to be shorter than bupivacaine alone and onset
time longer than chloroprocaine alone

205
Q
852. Each of the following is associated with an increased incidence postdural puncture headaches (PDPH)
EXCEPT
A. Young age
B. Female gender
C. Early ambulation
D. Pregnancy
E. Large needle size
A

852.
(C)
Younger adults have a higher incidence of PDPH than older adults. Women have a slightly higher incidence
than men. Pregnant women have a higher incidence than nonpregnant women. Since the incidence and
severity of PDPH relates to the amount of CSF leakage through the dural hole, it makes sense that the larger
the needle and the more holes in the dura the greater incidence of PDPH. In addition, the shape of the tip of
the needle is important; a cutting needle (e.g.. Quincke) has a greater incidence of PDPH than noncutting
needles (e.g., Whitacre, Sprotte). The incidence of headache has been shown to be less when the dural fibers
are split longitudinally rather than when they are cut while the needle is held in a transverse direction. The
timing of ambulation relative to dural puncture has not been shown to affect the incidence of postspinal
headache. The block should wear off before ambulation is attempted

206
Q
  1. Each of the following items describes pain in the abdominal viscera EXCEPT
    A. Pain is transmitted via the vagus nerve
    B. The nerve fibers are type C
    C. Pain is not in a dermatomal distribution
    D. Pain is characterized by a dull aching or burning sensation
    E. Distention of the transverse colon causes more pain than surgical transection
A

853.
(A)
Virtually all pain arising in the thoracic or abdominal viscera is transmitted via the sympathetic nervous system
in unmyelinated type C fibers. Visceral pain is dull, aching, burning, and nonspecific. Visceral pain is caused
by any stimulus that excites nociceptive nerve endings in diffuse areas. In this regard, distention of a hollow
viscus causes a greater sensation of pain than does the highly localized damage produced by transecting the
gut

207
Q
  1. A 24-year-old man undergoes repair of a right anterior shoulder dislocation under interscalene brachial
    plexus block. Anesthesia is produced with 30 mL of 0.5% bupivacaine with 5 μg/mL of epinephrine. The next
    morning, the patient complains of numbness in his right arm and hand. The most likely cause of these
    complaints is
    A. Excessive retraction by the surgeon
    B. Prolonged pressure on the brachial plexus from malpositioning
    C. Pressure on the right medial epicondyle from malpositioning
    D. Pressure on the right posterior humerus from malpositioning
    E. Residual anesthesia
A

854.
(E)
The brachial plexus is not normally retracted during repair of an anterior shoulder dislocation. Prolonged
pressure on the brachial plexus will result in hand or arm numbness. This may occur if this structure becomes
pinched between the clavicle and the head of the humerus, as seen in patients placed in steep Trendelenburg
position with the shoulders resting against shoulder braces. Prolonged pressure on the medial epicondyle may
produce an ulnar neuropathy, whereas prolonged pressure against the posterior surface of the humerus may
produce a radial neuropathy. Bupivacaine is a long-acting local anesthetic and may cause numbness for 8 to
12 hours

208
Q
  1. Which of the following patients would be LEAST likely to develop a decrease in heart rate with a high (C8)
    level spinal anesthesia?
    A. A 15-year-old female patient with history of Wolff-Parkinson-White syndrome
    B. A 73-year-old patient with glaucoma treated with pilocarpine eye drops
    C. A 33 year old with a T6 paraplegia
    D. A 45-year-old diabetic man with a history orthostatic hypotension
    E. A 47-year-old patient who had a myocardial infarction 1 month ago, now taking procainamide
A

855.
(D)
The cardiac accelerator fibers originate in the T1-T4 segments. A high spinal, above T1, can cause
bradycardia by anesthetizing these fibers. Diabetic patients who display orthostatic hypotension have an
autonomic neuropathy. The cardiac accelerator fibers are essentially ablated in these patients and therefore,
the slowing of heart rate does not ordinarily develop with high spinals. Pilocarpine, a parasympathomimetic
agent, will not prevent bradycardia with spinal anesthesia. Patients with Wolff-Parkinson-White syndrome will
become bradycardic when the autonomic accelerator fibers are interrupted, as will patients with a spinal cord
transection below T4. Recent myocardial infarction does not eliminate susceptibility to bradycardia with
sympatholysis unless the patient has a complete heart block

209
Q
  1. A 35-year-old woman receives a popliteal block for ankle and foot surgery. Which other nerve must be
    blocked in order to have complete anesthesia of the foot?
    A. Deep peroneal nerve
    B. Superficial peroneal nerve
    C. Sural nerve
    D. Saphenous nerve
    E. Posterior tibial nerve
A

856.
(D)
All of the nerves of the foot with the exception of the saphenous are derived from the sciatic nerve. The
sciatic nerve distally becomes the tibial and peroneal nerves which can be blocked at the popliteal fossa for
surgery below the knee. The saphenous nerve is a branch of the femoral nerve and provides sensory
innervation along the medial aspect of the lower leg between the knee and the medial malleolus and must
also be blocked for surgery below the knee

210
Q
857. The most common complication of a celiac plexus block is
A. Hypotension
B. Seizure
C. Subarachnoid injection
D. Retroperitoneal hematoma
E. Constipation
A

857.
(A)
The sympathectomy produced by a celiac plexus block causes hypotension by decreasing preload to the
heart. This complication can be avoided by volume loading the patient with lactated Ringer’s solution.
Subarachnoid injection is the most serious complication of celiac plexus block. Seizure is possible with an
intravascular injection. Retroperitoneal hematoma is also possible, but extremely rare. This block frequently
relieves constipation by interrupting the sympathetic fibers and leaving the parasympathetic fibers unopposed

211
Q
  1. The occipital portion of the skull receives sensory innervation from
    A. Spinal accessory nerve (nerve XI)
    B. Facial nerve (nerve VII)
    C. Ophthalmic branch of trigeminal nerve (nerve V)
    D. Maxillary branch of trigeminal nerve (nerve V)
    E. None of the above
A

858.
(E)
The occiput receives sensory innervation from the greater and lesser occipital nerves, which are terminal
branches of the cervical plexus. Blockade of these nerves is usually carried out as a diagnostic step in the
evaluation of head and neck pain

212
Q
859. Each of the following is a potential complication of lumbar sympathetic blocks EXCEPT
A. Puncture of the renal pelvis
B. Intravascular injection (aorta)
C. Seizure
D. S1 nerve block
E. Accidental subarachnoid injection
A

859.
(D)
Potential complications from lumbar sympathetic block include subarachnoid injection, puncture of a major
vessel (e.g., aorta) or renal pelvis, neuralgia, somatic nerve damage, perforation of a disk, infection,
ejaculatory failure, and chronic back pain. Blockade of nerves arising from the lumbar plexus is possible, but
given the anatomic location of the sacral plexus, blockade of an S1 nerve would be extremely unlikely if not
impossible

213
Q
  1. After placement of an epidural catheter in a 55-year-old patient for total hip arthroplasty, an entire epidural
    dose is administered into the subarachnoid space. Physiologic effects consistent with subarachnoid injection
    of large volumes of local anesthetic include all of the following EXCEPT
    A. Hypotension
    B. Bradycardia
    C. Respiratory depression
    D. Constricted pupils
    E. Possible cauda equina syndrome
A

860.
(D)
With the unintentional injection of an epidural dose of local anesthetic into the subarachnoid space, spinal
anesthesia develops rapidly. Blockade of the sympathetic fibers (T1-L2) produces hypotension, particularly if
the patient is hypovolemic. Bradycardia is produced by blocking the cardiac accelerator fibers (T1-T4).
Respiratory arrest is due to hypoperfusion of the respiratory centers as well as paralysis of the phrenic nerve
(C3-C5). The pupils become dilated after intrathecal injection of large quantities of local anesthetics; they will
return to normal size after the block recedes. Cauda equina syndrome has occasionally developed when the
epidural dose was unintentionally administered into the subarachnoid space (most commonly with
chloroprocaine). If one suspects an unintentional placement of the epidural dose subarachnoid, supportive
methods are initially done (the basic ABC’s of resuscitation). One can also aspirate CSF from the epidural
catheter (if it was inserted) to help remove some of the drug as well as reducing the pressure in the
subarachnoid space, which might help better perfuse the spinal cord and decrease the chance of cauda
equina syndrome developing

214
Q
  1. A 49-year-old type I diabetic patient with a long history of burning pain in the right lower extremity receives a
    spinal anesthetic with 100 mg of procaine with 5% dextrose. The patient reports no relief in symptoms but
    has complete bilateral motor blockade. What diagnosis is consistent with this differential blockade
    examination?
    A. Diabetic neuropathy
    B. Central pain
    C. Myofascial pain
    D. Meralgia paresthetica
    E. Complex regional pain syndrome I (reflex sympathetic dystrophy)
A

861.
(B)
Somatic pain in the extremities is relieved with spinal anesthesia. If a patient fails to obtain pain relief despite
complete sympathetic, sensory, and motor blockade, a “central” mechanism for the pain is likely or the lesion
causing the pain is higher in the CNS than the level of blockade achieved by the spinal. Central pain states
may include encephalization, psychogenic pain, or malingering. Persistence of pain in the lower extremities
after successful spinal blockade suggests a central source or psychological source of pain

215
Q
  1. An 18-year-old man has a seizure during placement of an interscalene brachial plexus block with 2%
    lidocaine. The anesthesiologist begins to hyperventilate the patient’s lungs with 100% O2 using an anesthesia
    bag and mask. The rationale for this therapy includes all of the following EXCEPT
    A. Helps to prevent and treat hypoxia
    B. Hyperventilation decreases blood flow and delivery of lidocaine to the brain
    C. Hyperventilation induces hypokalemia which elevates the seizure threshold
    D. Hyperventilation induces alkalosis which elevates the seizure threshold
    E. Hyperventilation induces alkalosis which converts lidocaine to the protonated (ionized) form
A

862.
(E)
During a seizure, administration of 100% O2 helps to prevent and treat hypoxia in a patient who otherwise
might not be breathing. Hyperventilation also causes cerebral vasoconstriction and decreased delivery of local
anesthetic to the brain. Hyperventilation induces hypokalemia and respiratory alkalosis, both of which result in
hyperpolarization of nerve membranes and elevation of the seizure threshold. Hyperventilation also raises the
patient’s pH (respiratory alkalosis) and converts lidocaine into the non-ionized (nonprotonated) form, which
crosses the membrane more easily than the ionized form, which is detrimental

216
Q
863. Para-aminobenzoic acid is a metabolite of
A. Mepivacaine
B. Ropivacaine
C. Bupivacaine
D. Procaine
E. Prilocaine
A

863.
(D)
Para-aminobenzoic acid is a metabolite of the ester-type local anesthetics. Local anesthetics may be placed
into two distinct categories based on their chemical structure: ester or amide. The amides, which are
ropivacaine lidocaine, etidocaine, prilocaine, mepivacaine and bupivacaine, are metabolized in the liver. The
ester local anesthetics are cocaine, procaine, chloroprocaine, tetracaine, and benzocaine. These drugs are
metabolized by the enzyme pseudocholinesterase found in the blood. Para-aminobenzoic acid is a metabolic
breakdown product of ester anesthetic and is responsible for allergic reactions in some individuals

217
Q
  1. Which statement concerning peripheral nerve structure and function is FALSE?
    A. Both nonmyelinated and myelinated nerves are surrounded by Schwann cells
    B. The speed of propagation of an action potential along a nerve axon is greatly enhanced by myelin
    C. Generation of an action potential is an “all-or-nothing” phenomenon
    D. Propagation of an action potential along myelinated nerve axons occurs by saltatory conduction via the
    nodes of Ranvier
    E. Myelination renders nerves less sensitive to local anesthetic blockade
A

864.
(E)
Peripheral nerve axons are always enveloped by a Schwann cell. The myelinated nerves may be enveloped
many times by the same Schwann cell. Transmission of nerve impulses (i.e., action potentials) along
nonmyelinated nerves occurs in a continuous fashion, whereas transmission along myelinated nerves occurs
by saltatory conduction from one node of Ranvier to the next. Myelination speeds transmission of neurological
impulses; it also renders nerves more susceptible to local anesthetic blockade. An action potential is
associated with an inward flux of sodium that occurs after a certain membrane threshold has been exceeded

218
Q
  1. A 42-year-old woman with a morbid fear of general anesthesia receives an interscalene block for shoulder
    arthroscopy consisting of 20 mL 0.5% ropivacaine. Much of her arm, shoulder and hand are numb, but the
    patient complains of pain as the incision is made at the upper portion of the shoulder. The most appropriate
    next step is
    A. Repeat block
    B. Perform intercostobrachial block
    C. Perform superficial cervical plexus block
    D. Perform a deep cervical plexus block
    E. Induce general anesthesia
A

865.
(C)
The needle insertion site for an interscalene block is C6. Local anesthetics usually spread to C5, C6 and C7
which supply much, but not all, of the cutaneous innervation to the shoulder. With low-to-moderate volume
blocks there will be sparing of the (C3-C4) nerve roots, which supply some of the innervation to the anterior
shoulder. Of note, C8 and T1 may also be spared, often resulting in the need for ulnar nerve supplementation
if this block were used for a hand operation. Complete anesthesia for shoulder arthroscopy may require a
supplemental superficial cervical plexus with use of low to moderate volumes of a local anesthetic

219
Q
  1. According to the 2004 American Society of Regional Anesthesia and Pain Medicine (ASRA) practice advisory
    on infectious complications of regional anesthesia and pain medicine, the most important action to maintain
    aseptic technique and prevent cross-contamination during regional anesthesia techniques is
    A. Wearing surgical gown
    B. Hand washing
    C. Using soap and water instead of alcohol-based antiseptics
    D. Keeping fingernails short
    E. Using povidone iodine (e.g., Betadine) instead of alcohol-based chlorhexidine to scrub
A

866.
(B)
Hand washing is one of the most important techniques to prevent infections especially when alcohol-based
antiseptic solutions are used with sterile gloves. Although soap and water remove bacteria, they do not
effectively kill organisms. Antiseptic solutions with alcohol appear better than nonalcoholic antiseptics (e.g.,
povidone iodine). Nail length does not appear to be a risk factor for infections, because the majority of
bacterial growth occurs along the proximal 1 mm of nail adjacent to the subungual skin. Universal use of
gowns and gloves does not appear to be better than gloves alone in preventing infections in ICUs and
presumably is less important than adequate hand washing and use of sterile gloves (Hebl JR: Infections
complications:

220
Q
  1. A 75-year-old woman with a history of pulmonary embolism is scheduled for a right lower lobectomy for lung
    cancer. She is receiving dalteparin (Fragmin) for deep vein thrombosis (DVT) prophylaxis. How long after her
    last dose should one wait prior to placement of a thoracic epidural?
    A. 12 hours
    B. 24 hours
    C. 30 hours
    D. 72 hours
    E. No waiting necessary
A

867.
(A)
In patients taking low-molecular weight heparin, or LMWH (e.g., enoxaparin, dalteparin, tinzaparin), caution
should be exercised before proceeding with an epidural or spinal anesthetic because of the risk of producing
an epidural or spinal hematoma. The amount of time between the last dose of the LMWH and the relative
safety of starting a central neuraxial block depends on the dose of the LMWH. At the lower doses, used for
thromboprophylaxis, the LMWH should be held at least 10 to 12 hours prior to the block. At the higher doses,
used to treat an established DVT, one should wait at least 24 hours after the last dose of LMWH prior to the
block

221
Q
868. How long should a patient be off clopidogrel (Plavix) before performing a central neuraxial block?
A. 1 day
B. 2 days
C. 7 days
D. 14 days
E. No waiting necessary
A

868.
(C)
Patients taking nonsteroidal anti-inflammatory drugs (NSAIDs), ticlopidine and clopidogrel, exert effects on
platelet function. Nonsteroidal anti-inflammatory drugs (NSAIDs) are not a problem if given alone before
epidural or spinal anesthesia. But patients taking ticlopidine should wait 14 days and patients taking
clopidogrel should wait 7 days before having a neuraxial block placed, because of the increased risk of spinal
hematoma formation. Keep in mind that caution is always needed and the ASRA statement of “Careful
preoperative assessment of the patient to identify alterations of health that might contribute to bleeding is
crucial” is important

222
Q
869. Addition of bicarbonate to local anesthetics results in
A. Delayed onset of action
B. Reduced toxicity
C. Increased duration of action
D. Increased anesthetic potency
E. Reduced pain with skin infiltration
A

869.
(E)
Adding sodium bicarbonate to local anesthetic solutions hastens the onset of action of the local anesthetics,
especially when the local anesthetic solution contains epinephrine (which is produced at a lower pH). By
raising the pH, more of the local anesthetic is in the non-ionized, more lipid-soluble state. Raising the pH too
much (i.e., >6.05-8) would cause precipitation of the local anesthetic. It also seems to decrease pain with skin
infiltration. Pain on injection can also be decreased by a slow injection of the local anesthetic

223
Q
870. Through which of the following would a spinal needle NOT pass during a midline placement of a subarachnoid block in the L3-L4 lumbar space?
A. Supraspinous ligament
B. Interspinous ligament
C. Ligamentum flavum
D. Posterior longitudinal ligament
E. Dura mater
A

870.
(D)
This figure shows the anatomic structures that must be traversed by the spinal needle during performance of
a subarachnoid block. The structures include the skin, subcutaneous tissue, supraspinous ligament,
interspinous ligament, the ligamentum flavum, and finally the dura (posterior). If you were to continue to
advance the spinal needle, you would encounter the dura (anteriorly) while exiting the subarachnoid space,
the posterior longitudinal ligament, the periosteum of the vertebral body, and finally, bone

224
Q
871. What epidural dose of bupivacaine will give similar sensory analgesia as 10 mL of 2% lidocaine?
A. 5 mL of 0.25%
B. 10 mL of 0.25%
C. 5 mL of 0.5%
D. 10 mL of 0.5%
E. 5 mL of 0.75%
A

871.
(D)
In the epidural space, bupivacaine is four times more potent than lidocaine, so 0.5% bupivacaine is similar to
2% lidocaine. The duration of the bupivacaine block will be longer because bupivacaine has a long duration of
action and lidocaine has an intermediate duration of action

225
Q
  1. Each of the following additives to a spinal anesthetic possesses analgesic properties EXCEPT
    A. Clonidine
    B. Neostigmine
    C. Epinephrine
    D. Fentanyl
    E. All of the above posses analgesic activity
A

872.
(E)
Drugs with α-agonist activity (phenylephrine/epinephrine) possess some analgesic activity but less than
narcotics and local anesthetics. In addition, intrathecal epinephrine will reduce systemic/vascular uptake of
local anesthetics, thereby enhancing their effects, including hypotension. Clonidine alone, when administered
neuraxially, is an effective analgesic. Neostigmine has some mild analgesia properties but experience is
limited. Opioids added to the spinal solution enhance surgical anesthesia and provide postoperative pain.
Fentanyl 25 μg is commonly added for short surgical procedures (outpatient) whereas morphine can be used
when longer postoperative analgesia is desired for inpatients

226
Q
873. Which of the following local anesthetics is inappropriately paired with a clinical application because of its
properties or toxicity?
A. Tetracaine, topical anesthesia
B. Bupivacaine, intravenous anesthesia
C. Prilocaine, infiltrative anesthesia
D. Chloroprocaine, epidural anesthesia
E. Ropivacaine, epidural anesthesia
A

873.
(B)
For topical anesthesia, lidocaine, tetracaine, dibucaine and benzocaine are effective, as well as the
combination of lidocaine and prilocaine or EMLA cream. For intravenous regional anesthesia or Bier blocks,
many drugs have been used. Ester local anesthetics are not used for IV regional blocks because they can be
broken down in the blood stream (by plasma ester hydrolysis) which can shorten the drug’s duration of action
and can also cause thrombophlebitis of the vein (reported with chloroprocaine). Because cardiovascular
collapse has been reported with bupivacaine, and would likely also occur with etidocaine and ropivacaine, if
the tourniquet unintentionally is released while the block is setting up, they are not used for intravenous
regional anesthesia. Lidocaine and prilocaine are used for Bier blocks because of their relative safety. For
infiltrative anesthesia, all local anesthetics can be used. All local anesthetics can be used in the epidural
space, although procaine and tetracaine are rarely used (procaine has a slow onset and tetracaine has
marked motor block)

227
Q
  1. Discharge criteria from the post-anesthesia care unit (PACU) would be reached fastest after a 20 to 30 mL
    volume of which of the following epidurally administered local anesthetics?
    A. 3% 2-chloroprocaine
    B. 2% lidocaine
    C. 1% etidocaine
    D. 0.75% ropivacaine
    E. 0.5% levobupivacaine
A

874.
(A)
Procaine and 2-chloroprocaine have a short duration of action; lidocaine, mepivacaine and prilocaine have an
intermediate duration of action; etidocaine, bupivacaine, levobupivacaine, tetracaine and ropivacaine have a
long duration of action. For similar sensory anesthesia, a higher concentration of local anesthetic is needed for
the short duration of local anesthetics compared with both the intermediate and long duration agents, because
they are less potent

228
Q
  1. A caudal block with 0.25% bupivacaine and 1:200,000 epinephrine is planned for postoperative analgesia
    after bilateral inguinal hernia repair in a 5 month old. Each of the following would be consistent with an
    intravascular injection EXCEPT
    A. Systolic blood pressure increase by 15 mm Hg
    B. Heart rate decrease by 10 bpm
    C. Ventricular extrasystoles
    D. Seizure
    E. T-wave amplitude of 25 percent over baseline
A

875.
(B)
A change in the T-wave amplitude of 25 percent, an increase in heart rate of 10 beats per minutes, or systolic
blood pressure greater than 15 mm Hg is considered a positive response to an epinephrine containing local
anesthetic solution. A slight drop in heart rate may result if the block is properly performed and no
intravascular injection occurs

229
Q
876. Which is NOT a potential complication of a stellate ganglion block
A. Recurrent laryngeal nerve paralysis
B. Subarachnoid block
C. Brachial plexus block
D. Pneumothorax
E. Increased heart rate
A

876.
(E)
All of the choices listed are potential complications of stellate ganglion blockade except an increase in heart
rate. The stellate ganglion supplies sympathetic fibers to the upper extremity and head and some to the heart.
Loss of the cardiac acceleratory fibers may slow the heart rate, not speed it up. Other potential complications
of stellate ganglion blockade include accidental injection of the local anesthetic into a vertebral artery resulting
in seizure and inadvertent cervical epidural

230
Q
  1. An axillary block utilizing the transarterial approach with 0.5% bupivacaine and epinephrine (1:200,000) is
    performed in a 70-kg patient. Thirty mL is injected posterior to the axillary artery and 30 mL anterior to it.
    How many mg have been injected and was the maximum recommended dose exceeded?
    A. 150 mg bupivacaine, 150 μg epinephrine did not exceed maximum dose
    B. 150 mg bupivacaine, 150 μg epinephrine exceeded maximum dose
    C. 300 mg bupivacaine, 300 μg epinephrine did not exceed maximum dose
    D. 300 mg bupivacaine, 300 μg epinephrine exceeded maximum dose
    E. Transarterial blocks should never contain epinephrine and the block should not be done
A

877.
(D)
A total of 60 mL of 0.5% bupivacaine with epinephrine (1:200,000) was used. A 0.5% solution = 0.5 g in 100
mL of fluid = 500 mg/100 mL = 5 mg/mL. A 1:200,000 solution means 1 gram in 200,000 mL = 1000
mg/200,000 mL = 1 mg/200 mL = 1000 μg/200 mL = 5 μg/mL. Therefore 60 mL of 0.5% bupivacaine with
1:200,000 epinephrine contains 60 mL × 5 mg/mL or 300 mg bupivacaine and 60 mL × 5 μg/mL or 300 μg of
epinephrine. For a major nerve block the maximum recommended dose with epinephrine (1:200,000) is 500
mg for lidocaine and mepivacaine, 600 mg with prilocaine, and 225 mg with bupivacaine. Epinephrine is used
in the local anesthetic to check for intravascular injection of the incremental doses and is not contraindicated
but should be used for this block

231
Q
  1. Five days after knee arthroscopy under spinal anesthesia, a 55 year old complains of double vision and
    difficulty hearing. The other likely finding would be
    A. Headache
    B. Fever
    C. Weakness in legs
    D. Mental status changes
    E. Backache
A

878.
(A)
Post dural puncture headaches (spinal headaches) usually develop within 12 to 48 hours after a dural
puncture, but may develop immediately or take months to develop. The most characteristic symptom is a
postural component where the headache occurs in the upright position and is usually completely gone when
the patient is in the supine position. The headache is typically frontal and/or occipital in location. Other
symptoms include nausea, vomiting, anorexia, visual disturbances (blurred vision, double vision, photophobia)
and occasionally hearing loss (routinely found with auditory testing)

232
Q
879. Administration of an interscalene block is associated virtually 100% of the time with
A. Hoarseness
B. Ulnar nerve blockade
C. Ipsilateral Horner’s syndrome
D. Diaphragmatic hemiparalysis
E. Bradycardia
A

879.
(D)
Ipsilateral phrenic nerve block with diaphragmatic paralysis occurs is virtually 100% of patients receiving an
interscalene block. This produces a 25% reduction in pulmonary function, making this block a contraindication
in patients with borderline pulmonary function. Blockage of the recurrent laryngeal nerve can occur but is rare;
however, if the patient has contralateral vocal cord palsy and develops a recurrent laryngeal nerve block,
complete airway obstruction can occur. With this block, the inferior trunk of the brachial plexus where the ulnar
nerve is derived may be spared

233
Q
880. Which of the following nerves can be electrically stimulated at the ankle to produce flexion of the toes?
A. Posterior tibial nerve
B. Saphenous nerve
C. Deep peroneal nerve
D. Superficial peroneal nerve
E. Sural nerve
A

880.
(A)
Five nerves are blocked when performing an ankle block. The saphenous, superficial peroneal, and sural
nerves are all sensory below the ankle and electrical stimulation would have no effect. Stimulation of the
posterior tibial nerve causes flexion of the toes by stimulating the flexor digitorum brevis muscles and
abduction of the first toe by stimulating the abductor hallucis muscles. The posterior tibial nerve also is
sensory to most of the plantar part of the foot. Stimulation of the deep peroneal nerve causes extension of the
toes by stimulating the extensor digitorum brevis muscles. The deep peroneal nerve has a small sensory
branch for the first interdigital cleft. From the practical standpoint, many anesthesiologists perform a purely
infiltration block of these nerves. If a nerve stimulator is used, it is mainly used to find the posterior tibial
nerve, which can be hard to anesthetize if small volumes of local anesthetic are administered. The posterior
tibial nerve can be difficult to stimulate in diabetics with diabetic neuropathy

234
Q
  1. Which of the following observations, after nerve injury, is correctly paired with the appropriate nerve?
    A. Inability to flex the forearm—ulnar nerve
    B. Numbness in the thumb—radial nerve
    C. Inability to extend the forearm—musculocutaneous nerve
    D. Numbness in the little finger—median nerve
    E. All are correctly paired
A

881.
(B)
The motor responses include: arm flexion at the elbow (musculocutaneous nerve), arm extension at the elbow
(radial nerve), forearm pronation, wrist flexion and thumb opposition (median nerve), ulnar deviation of the
wrist, little finger flexion, thumb adduction and flaring of the fingers (ulnar nerve), wrist and finger extension
(radial nerve). The sensory response (includes some variations) is: back of the arm, forearm and radial side
dorsal side of the hand (radial nerve), skin of the lateral forearm (musculocutaneous nerve), ulnar side of the
hand and both surfaces of the ulnar one and one-half fingers (ulnar nerve), the radial side of the palm of the
hand as well as the dorsal aspect of the radial three and one-half fingers (median nerve). To evaluate the
setup of a brachial plexus block, a common technique is to perform the four P’s (Push, Pull, Pinch, Pinch).
Have the patient push or extend the forearm (radial), pull or flex the forearm (musculocutaneous nerve), pinch the index or second finger (median nerve), pinch the little finger (ulnar nerve)

235
Q
  1. During an airway exam, a 53-year-old patient mentions that his right thumb tingles and then becomes numb
    if he extends his head for more than a few seconds. This symptom most likely represents a (an)
    A. Unstable c-spine
    B. Lhermitte’s phenomenon
    C. C6 nerve root irritation
    D. C8 radiculopathy
    E. Carpal tunnel syndrome
A

882.
(C)
Unilateral numbness or paresthesia in the upper extremity during extension of the neck usually represents
nerve root impingement at the vertebral foramina. Specifically, unilateral degenerative changes restrict the
foramen to such a degree that it compresses and irritates the nerve root traversing the vertebral foramen
when the head is extended. Treatment ranges from NSAIDs, to steroids and may require surgical intervention
if there is muscle weakness

236
Q
  1. When performing an interscalene block with a peripheral nerve stimulator you note diaphragmatic movement.
    You should now
    A. Inject the local anesthetic as the needle is in an appropriate location
    B. Redirect the needle in an anterior direction
    C. Redirect the needle in a more cephalad direction
    D. Redirect the needle in a posterior direction
    E. Advance the needle about 0.5 cm more and inject
A

883.
(D)
Although a successful interscalene block causes ipsilateral phrenic nerve paralysis in almost 100% of patients,
identifying the phrenic nerve means you are anterior to the brachial plexus and you should reposition your
needle. You should redirect the needle in a posterior direction

237
Q
884. During placement of an interscalene block the patient becomes hypotensive, bradycardic and cyanotic. Most
likely cause is
A. Vertebral artery injection
B. Carotid artery injection
C. Phrenic nerve blockade
D. Total spinal
E. Stellate ganglion block
A

884.
(D)
With an intravascular injection, the main symptoms would most likely be CNS toxicity (e.g., seizures) as blood
flow is directly to the brain. The Bezold-Jarish reflex (hypotension and bradycardia) has been reported in
awake, sitting patients undergoing shoulder surgery with an interscalene block. This maybe related to
intracardiac mechanoreceptors being stimulated by the decreased venous return in the sitting position. This
leads to decreased sympathetic tone and increased parasympathetic tone. Breathing is still present with this
reflex. Block of the stellate ganglion would produce a Horner’s syndrome, which is not associated with
breathing abnormalities. Injection into the intrathecal space is uncommon, but possible, especially if the needle
is not pointed in the caudal direction, and would lead to a total spinal block with little local anesthetic injected
(e.g., hypotension, bradycardia respiratory paralysis that would lead to cyanosis)

238
Q
885. The reason that ropivacaine is marketed as pure S enantiomers is because the S form is associated with
A. More rapid onset
B. Longer duration
C. Reduced cardiac toxicity
D. Reduced incidence of anaphylaxis
E. Less motor blockade
A

885.
(C)
The pipecoloxylidide local anesthetics (mepivacaine, bupivacaine, ropivacaine and levobupivacaine) are chiral
drugs, which means they have an asymmetric carbon atom (i.e., have a left- or S and a right- or R hand
configuration). Mepivacaine and bupivacaine are racemic mixtures (50% S: 50% R mixture). The pure S forms
show reduced neurotoxicity and reduced cardiotoxicity (e.g., ropivacaine and levobupivacaine). Lidocaine is an
achiral compound (i.e., has no chiral carbon atom)

239
Q
886. Nerves that originate from the sacral plexus include each of the following EXCEPT
A. Femoral nerve
B. Tibial nerve
C. Superficial peroneal nerve
D. Deep peroneal nerve
E. Sural nerve
A

886.
(A)
Nerves to the lower extremity emerge from the L2 to S3 nerve roots. The upper roots (mainly L2-L4) form the
lumbar plexus which gives rise to the femoral, obturator, and lateral femoral cutaneous nerves. A branch from
the lumbar plexus along with the sacral plexus gives rise to the sciatic nerve. Branches of the sciatic nerve
include the common peroneal (branches to make the superficial and deep) and the tibial, and the sural nerves

240
Q
887. The only technique shown to prevent anesthetic-related nerve injury (ARNI) during placement of peripheral
nerve blocks is
A. Ultrasound guided regional technique
B. Transarterial technique
C. Nerve stimulator
D. Paresthesia technique
E. None of the above
A

887.
(E)
Anesthetic-related nerve injuries to the brachial plexus are rare and poorly understood. The only way to
minimize nerve injury is to minimize trauma to neural fibers. Although ultrasound-guided technique is
promising, currently there is no clinical evidence for this

241
Q
  1. An axillary block is performed on a healthy 19-year-old athlete. Thirty mL of 0.75% bupivacaine is injected
    incrementally. Fifteen minutes after the bupivacaine injection, the patient has a seizure and experiences a
    ventricular fibrillation arrest. Which of the measures below is NOT indicated.
    A. Begin chest compressions at 100 per minute
    B. Ventilate with 100% oxygen
    C. Bolus propofol to bind local anesthetic
    D. Infuse 20% lipid emulsion
    E. Repeat emulsion infusion if fibrillation continues
A

888.
(C)
Local anesthetic toxicity is a multisystem phenomenon, but the most crucial manifestation involves the heart
(atrioventricular conduction block, arrhythmias, myocardial depression, and cardiac arrest). In isolated cardiac
tissue hypercarbia, acidosis and hypoxia will augment the negative inotropic and chronotropic effects of
bupivacaine. In the event of seizure and ventricular fibrillation, hypercarbia and hypotension would cause a
severe acidosis and hypoxia, which would potentiate the toxicity of bupivacaine. To reduce these cardiotoxic
effects, the initial resuscitation efforts are aimed at maximizing oxygen delivery, increasing tissue perfusion
and ventilation. This is accomplished through chest compressions, assisted ventilation with oxygen,
intravenous sodium bicarbonate, and inotropic and chronotropic support with pharmacological agents such as
atropine, epinephrine, dopamine, and dobutamine. Standard cardiopulmonary resuscitation may not be
successful in some cases, and placement of the patient on cardiopulmonary bypass may be necessary.
Recently, lipid infusions have been shown to reduce the toxicity of intravascularly injected bupivacaine.
Propofol is formulated as a lipid emulsion, and as such would bind bupivacaine to some degree, but this effect
would be overshadowed by the substantial cardiac depressant of the anesthetic

242
Q
889. The structure LEAST likely to be encountered during placement of interscalene block is
A. Phrenic nerve
B. Vertebral artery
C. Recurrent laryngeal nerve
D. Subarachnoid space
E. Vagus nerve
A

889.
(E)
When performing an interscalene block, the needle is usually placed at the line extending lateral to the cricoid
cartilage that intersects the interscalene groove at the C6 level. The needle in inserted in a slightly posterior
and a 45° caudad direction. The caudad direction is used to decrease the chance of injecting the local
anesthetic into the vertebral artery, or obtaining a spinal or epidural block. The phrenic nerve is routinely
blocked (100% of the time) and occasionally the recurrent laryngeal nerve is blocked

243
Q
890. All of the following are symptoms of a developing epidural hematoma EXCEPT
A. Radicular back pain
B. Bowel and bladder dysfunction
C. Sensory deficits
D. Motor deficits
E. Fever and chills
A

890.
(E)
Epidural hematomas are rare complications of spinal anesthesia (1:200,000) and epidural anesthesia
(1:150,000). However, in the presence of LMWH the incidence is much higher: 1:40,000 with spinal anesthesia
and 1:3000 with continuous epidural catheter. Clinical symptoms include radicular back pain, bowel and
bladder dysfunction, sensory or motor deficits. An MRI is the diagnostic test of choice and prompt
decompressive laminectomy is the treatment of choice

244
Q
891. In addition to C nerve fibers, which nerve fibers carry pain impulses?
A. A-α
B. A-β
C. A-γ
D. A-δ
E. B
A

891.
(D)
Peripheral nerves are classified according to the fiber size and physiologic properties such as the presence or
absence of myelin, conduction velocity, location, and function. Type A fibers range in diameter from 1 to 22
μm, are myelinated, and have moderate-to-fast conduction velocities. These fibers are subclassified into four
groups based on their location and function. Type A-alpha and A-beta fibers provide motor and proprioception
function to muscles and joints; type A-gamma fibers innervate muscle spindles and provide for muscle tone;
and type A-delta fibers provide pain, temperature and touch sensation. Type B fibers are preganglionic
sympathetic nerves that are less than 3 μm in diameter, myelinated, and have medium conduction velocities.
Type C fibers are postganglionic sympathetic nerves that are very small in diameter, are not myelinated, and
have slow conduction velocities. Type C fibers are also afferent sensory nerves involved in pain, temperature,
and touch

245
Q
  1. An intradural mass lesion at the tip of a drug infusion catheter is LEAST likely to present as
    A. Piloerection, rhinorrhea, sweating, and more pain
    B. Development of numbness in T8 dermatomal pattern
    C. Hypopnea
    D. Perianal numbness
    E. Unilateral hip flexor weakness
A

892.
(C)
Overdose of intrathecal opiates would not be a sign of an intradural mass lesion. Granulomas at the tip of
intrathecal catheters used with intrathecal drug delivery systems are gaining increased attention. Granulomas
are more frequently associated with high concentrations and doses of either morphine (>10 mg/day) or
hydromorphone (>10 mg/day). Most patients who will develop granulomas receive the intrathecal medications
for more than 6 months. Presenting symptoms may include loss of drug effect, new pain or paresthesias or
neurologic deficits. Patients should be routinely screened for signs and symptoms of granuloma formation at
scheduled intrathecal pump refill appointments. Suspicious cases should undergo prompt diagnostic imaging
and consideration of neurosurgical consultation

246
Q
  1. Benzocaine is unique among the local anesthetics for which of the following reasons?
    A. It is a weak acid
    B. It can be used topically
    C. It is metabolized by same enzyme as succinylcholine
    D. It can promote formation of methemoglobin
    E. Is a vasoconstrictor
A

893.
(A).
In addition to benzocaine, tetracaine and lidocaine can also be used as topical anesthetics.
Pseudocholinesterase, the enzyme responsible for the metabolism of succinylcholine, metabolizes all of the
ester local anesthetics, benzocaine, procaine, chloroprocaine and tetracaine. Benzocaine does promote the
formation of methemoglobin, but is not alone in that regard since prilocaine also causes formation of
methemoglobin. The pKa of benzocaine is 3.5, which qualifies it as a weak acid and as such exists in
uncharged at physiologic pH. All other local anesthetic pKa’s are higher than 7.4, meaning that some fraction
of them exists in the protonated form

247
Q
  1. Which statement concerning local anesthetics is CORRECT?
    A. The un-ionized form of a local anesthetic binds to the nerve membrane to actually block conduction
    B. If one node of Ranvier is blocked, conduction will be reliably interrupted
    C. The ability of a local anesthetic to block nerve conduction is directly proportional to the diameter of the
    fiber
    D. The presence of myelin enhances the ability of a local anesthetic to block nerve conduction
    E. Local anesthetics block transmission by inhibiting the voltage-gated potassium ion channels
A

894.
(D)
The un-ionized form of the local anesthetic traverses the nerve membrane whereas the ionized form actually
blocks conduction. About three nodes of Ranvier must be blocked to achieve anesthesia. The ability of a local
anesthetic to block conduction is inversely proportional to the diameter of the fiber. The presence of myelin
enhances the ability of a local anesthetic to block conduction, as does rapid firing. The local anesthetic blocks
nerve transmission by inhibiting the voltage-gated sodium ion channels

248
Q
  1. Postdural puncture headaches
    A. Usually occur immediately following dural puncture
    B. Are relieved 8 to 12 hours after an epidural blood patch is performed
    C. Occur more frequently in nonpregnant patients compared with pregnant patients
    D. Can be associated with neurologic deficits
    E. Are more frequent in the elderly compared with younger adults
A

895.
(D)
Postdural puncture headaches (PDPHs) typically appear within 12 to 48 hours of a dural puncture but may be
immediate and occasionally have become delayed for several days or months after a dural puncture. The
headaches are characterized by dull or throbbing frontal or occipital pain, which worsens with sitting and
improves with reclining. Postspinal headaches may be associated with neurologic symptoms such as diplopia,
tinnitus, and reduced hearing acuity. Very rarely, a subdural hematoma will develop. The etiology of postspinal
headaches is believed to be caused by a reduction in CSF pressure and resulting tension on meningeal
vessels and nerves (which results from leakage of CSF through the needle hole in the dura mater). Factors
associated with an increased incidence of postspinal headaches include pregnancy, size and type of needle
used to perform the block (larger needles and Quincke more common than smaller needles and Whitacre or
Sprotte), and the number of dural punctures. They occur more frequently in young adults compared with
children and the elderly. Conservative therapy for a postspinal headache includes bed rest, analgesics, and
oral and intravenous hydration. If conservative therapy is not successful after 24 to 48 hours, an epidural
“blood patch” with 10 to 20 mL of the patient’s blood can be performed. An epidural “blood patch” usually
provides prompt relief of the postspinal headache

249
Q
  1. Which of the following procedures for treatment of chronic pain requires localization of the epidural space with
    an epidural needle as part of technique?
    A. Transcutaneous electrical nerve stimulation
    B. Nucleoplasty
    C. Spinal cord stimulation
    D. Intradiskal electrothermal therapy
    E. Annuloplasty
A

896.
(C)
When inserting a spinal cord stimulator, a 15 gauge needle is advanced into the epidural space via the
paramedian approach. After confirmation of proper needle placement with AP and lateral fluoroscopic views,
the stimulation electrode is passed through the needle and threaded to the desired vertebral level. The needle is then removed and the leads attached to the external programmer. Transcutaneous electrical nerve
stimulation (TENS) unit electrodes are applied to the skin. With nucleoplasty, intradiskal electrothermal therapy
and annuloplasty, the needle is inserted to access the intervertebral disk

250
Q
  1. Each of the following drugs has been used to treat neuropathic pain. Selective inhibition of serotonin and
    norepinephrine reuptake is the mechanism of which drug?
    A. Duloxetine
    B. Mexiletine
    C. Gabapentin
    D. Tramadol
    E. Carbamazepine
A

897.
(A)
Many drugs have been used to treat neuropathic pain, including analgesics (NSAIDs and opioids), firstgeneration
antiepileptic drugs (e.g., carbamazepine and phenytoin), second-generation antiepileptic drugs
(e.g., gabapentin, pregabalin), topical agents (e.g., lidocaine, capsaicin), antiarrhythmics (e.g., mexiletine),
tricyclic antidepressants (e.g., amitriptyline, nortriptyline, desipramine) as well as other antidepressants (e.g.,
duloxetine, venlafaxine). Duloxetine (Cymbalta) is a selective serotonin and norepinephrine reuptake inhibitor
(SNRI) that is used for major depressive disorders, generalized anxiety disorders, fibromyalgia and diabetic
peripheral neuropathic pain. Mexiletine is an orally effective amine analogue of lidocaine and may be effective
in decreasing neuropathic pain when other drugs have failed. Gabapentin, a structural analogue of GABA,
works by increasing the synthesis of the inhibitory neurotransmitter GABA. Tramadol is an opioid agonist that
has moderate activity at the μ receptor and weak activity at the κ and δ opioid receptors. In addition, tramadol
enhances the spinal inhibitory pathways by inhibiting the neuronal uptake of norepinephrine and serotonin as
well as presynaptic stimulation of serotonin release. Carbamazepine (Tegretol) is an anticonvulsant with
specific analgesic properties for trigeminal neuralgia. Carbamazepine seems to reduce polysynaptic responses
by an unknown mechanism

251
Q

Picture: Hall page 18

  1. Musculocutaneous nerve
  2. Axillary artery
  3. Axillary vein
  4. Ulnar nerve
A
  1. (E)
  2. (B)
  3. (A)
  4. (C)
252
Q
  1. Phrenic nerve
  2. Cardiac accelerator fibers
  3. Pudendal nerve
  4. Pain fibers to the uterus
  5. Inhibitory presynaptic fibers to the gastrointestinal tract
    A. C3-C5
    B. T1-T4
    C. T5-T12
    D. T10-L1
    E. S2-S4
A
  1. (A)
  2. (B)
    904 (E)
  3. (D)
  4. (C)
    In the normal adult, breathing and coughing can be done exclusively by the diaphragm, which is innervated by
    the phrenic nerve (C3-C5). The heart rate is dependent upon intrinsic pacemaker activity of the SA node,
    which can be affected by the autonomic nervous systems sympathetic nervous systems cardiac accelerator
    fibers (T1-T4) as well as the parasympathetic nervous systems vagus nerve (cranial nerve 10). The first stage
    of labor pain is related to uterine contractions and dilation of the cervix (T10-L1). The second stage of labor is
    related to both uterine pain (T10-L1), as well as birth canal pain which is supplied by the pudendal nerves
    (S2-S4). The greater splanchnic (T5-T9) and the lesser splanchnic (T10-T12) nerves supply sympathetic
    fibers to the celiac plexus which inhibits much of the gastrointestinal tract
253
Q
  1. Sensory innervation to the posterior one third of the tongue
  2. Motor innervation to the cricothyroid muscle
  3. Sensory innervation below the vocal cords to the carina
  4. Sensory innervation to the mucous membranes of the false cords
  5. Motor innervation to the tonsils
  6. Sensory innervation to the posterior pharynx
  7. Motor innervation to most of the intrinsic muscles of the larynx
  8. Motor innervation to the superior and middle constrictor muscles
    A. Glossopharyngeal nerve
    B. Internal branch of the superior laryngeal nerve
    C. External branch of the superior laryngeal nerve
    D. Recurrent laryngeal nerve
    E. Cranial nerve XI
A
  1. (E)
  2. (C)
  3. (D)
  4. (B)
  5. (A)
  6. (A)
  7. (D)
  8. (E)
    When an awake intubation is needed, local anesthetics can be applied topically or by injection. Innervation of
    the airway includes the glossopharyngeal nerve, cranial nerve XI, and branches of the vagus nerve (internal
    and external branches of the superior laryngeal nerve and the recurrent laryngeal nerve). The
    glossopharyngeal nerve provides sensory innervation of the posterior one third of the tongue, the vallecula
    and the anterior surface of the epiglottis (lingual branch), the pharyngeal walls (pharyngeal branch) and the
    tonsils (tonsillar branch). With the exception of the cricothyroid muscle, the recurrent laryngeal nerve of the
    vagus provides motor innervation of all the intrinsic muscles of the larynx. The cricothyroid muscle is supplied
    by the external branch of the superior laryngeal nerve of the vagus. The sensory innervation of the mucosa of
    the larynx down to the vocal folds comes from the internal branch of the superior laryngeal nerve of the
    vagus, and the sensory innervation of the mucosa of the larynx below the vocal folds comes from the
    recurrent laryngeal nerve of the vagus. The muscles of the pharynx are supplied through the pharyngeal
    plexus from motor fibers from the eleventh cranial nerve (accessory nerve)